Treating late-life decline: When more is less

Article Type
Changed
Tue, 12/11/2018 - 15:32
Display Headline
Treating late-life decline: When more is less

History: A fading memory

Mrs. S, 85, lives alone in her home of 40 years. Over the past 3 years, she has complained increasingly about headaches, fatigue, and back pain. The cause of these vague physical difficulties has not been determined.

Her daughters say that Mrs. S has become increasingly forgetful. She often does not remember family visits, has difficulty organizing her bank accounts, repeatedly misplaces her pocketbook, and on one occasion became lost on her way to the supermarket. Once fairly social, she has become increasingly isolated.

How would you address Mrs. S’ impaired memory? What medical or psychiatric problems might her forgetfulness indicate?

Dr. Verma’s observations

This case illustrates the fundamentals of geriatric care, the first of which is to preserve—if not enhance—the patient’s function (Box). Forty years ago, Kral1 identified “benign senescent forgetfulness” as a normal aspect of aging. Current research, however, suggests that “senescent forgetfulness” is not always benign. Alexopoulos, Krishnan, and others2,3 have shown that depression manifesting in late life is accompanied by significant white-matter change and substantially increases the risk of developing dementia.

Well past the point of isolated forgetfulness, Mrs. S is exhibiting functional decline and cognitive impairment in multiple domains. The question is, are these symptoms the result of a medical problem such as Alzheimer’s disease or dementia, a psychiatric disorder, or both? The workup and management of these complaints can dramatically affect subsequent outcomes.

Vague medical complaints of unknown cause should not necessarily imply that the problem is psychiatric. The clinician should rule out common medical causes of cognitive decline, including:

  • drug toxicity, especially after anxiolytic and sedative-hypnotic agents have been administered
  • endocrine dysfunction, such as hypothyroidism
  • and CNS neoplasms (Table 1).

Box

10 FUNDAMENTALS OF GERIATRIC CARE

  1. Preserve—if not enhance—the patient’s function
  2. Respect equilibrium; even apparently simple drug interventions can unintentionally upset a precarious adaptive reserve and trigger functional decline
  3. Beware of drug-drug interactions; most older persons are taking multiple medications
  4. Start low and go slow, but do not stop at subtherapeutic dosages
  5. Carefully consider a psychotropic’s side-effect profile
  6. Nondrug interventions are almost always safer than drugs
  7. Drugs do not replace compassion and caring
  8. Two (or more) drugs are not better than one
  9. One drug does not fit all patients
  10. Primum non nocere

A detailed history (still the best diagnostic procedure), a thorough physical evaluation, and routine lab tests can usually help rule out most of these causes. On the other hand, affirmative diagnosis when psychiatric symptoms are evident can minimize testing that can be emotionally, physically, and financially draining to the patient. In Mrs. S’ case, the prominence of the cognitive decline and attendant social withdrawal clearly point to depression or dementia.

The significant overlap between depression and dementia further complicates the diagnosis. Neuropsychological testing can uncover distinguishing factors, but it may help to empirically consider that all late-life depression with cognitive impairment may be secondary to early dementia.

Initiating early drug treatment of dementia with a cholinesterase inhibitor such as donepezil, galantamine, or rivastigmine may slow the trajectory of decline. Vascular risk factors—hypertension and diabetes in particular—also need to be controlled. Low-dose aspirin may help prevent microembolic phenomena.

A selective serotonin reuptake inhibitor (SSRI) can alleviate the depression. If mild paranoia is noted, adding an atypical antipsychotic at a low dosage (olanzapine, 1.25 to 5 mg once daily or risperidone, 0.5 to 1.0 mg/d divided in two doses) may help.

Above all, encourage the patient to remain physically and mentally active. To this end, the clinician should enlist the family and other caregivers to help motivate the patient. Involvement in a day program or similar program may alleviate the patient’s social isolation.

Treatment: New surroundings

Since her initial evaluation 1 year ago, Mrs. S reluctantly has moved into an assisted living facility at her daughters’ insistence. She adjusted well—at least for the first month or so. She then starting calling her daughters at all hours, complaining of being alone and scared. She was taken to a new internist, who prescribed oxazepam, 15 mg bid, for an “anxiety disorder.”

Instead of adjusting to her new surroundings, Mrs. S began to withdraw further. She stayed in her room most days, not even venturing to the dining room for meals. Her personal hygiene deteriorated. According to staff reports, “Mrs. S did not mix with the other residents,” and was becoming “increasingly paranoid.” Her calls to her family had escalated into bitter complaints that people were stealing her belongings.

Table 1

COMMON CAUSES OF DEMENTIA

ReversibleIrreversible
DepressionAlzheimer’s-type dementia
DeliriumVascular dementia
Drug toxicityLess common causes
 Anxiolytics Lewy body dementia
 Sedative-hypnotic drugs Parkinsonian dementia
  Pick’s disease

Mrs. S again visited the internist who, upon hearing that the patient was becoming more paranoid, assumed that she was exhibiting psychotic features. The internist diagnosed Mrs. S as having late-onset Alzheimer’s-type dementia with delusions and added haloperidol, 0.5 mg tid, to her regimen.

 

 

How would you help Mrs. S adjust to her new surroundings? How would your treatment plan differ from that of the internist?Table 2

ANTIPSYCHOTICS: SIDE-EFFECT PROFILES

 Haloperidol 0.75-2 mg/dClozapine 25-100 mg/dRisperidone 0.5-2 mg/dOlanzapine 5-15 mg/dQuetiapine 25-700 mg/d*Ziprasidone 40-160 mg/d*
Anticholinergic effects+/-++----
Sedation+++++++++++
Extrapyramidal symptoms++++-+---
Tardive dyskinesia++---??
Hypotension++++++-+-
QTc prolongation+?---+
Weight gain+++++++++/-
Diabetes mellitus+++++?
* Side effects are probably more severe at higher dosages, but the data are not clear.

Dr. Verma’s observations

The decision to transfer a loved one to a nursing home is difficult for all concerned. I have often seen caregiver “burnout” play a major role in the family’s decision.

After 40 years in her own home, Mrs. S is not likely to adjust readily to living in a “regimented” environment, no matter how comfortable and elegant it may seem. The phenomenon is often called “transfer trauma” and manifests as a sharp decline in function upon moving to a new environment. Most individuals do adapt with time; involving Mrs. S. in a socialization program and insisting on her presence during meals and at other facility events would have hastened her adjustment. Above all, clinicians should be supportive and avoid resorting to medication too soon.

Because Mrs. S’ functional decline was so sharp, however, trying a nondrug therapy would have been easier said than done. Indeed, the internist resorted too quickly to medication, prescribing a short-acting benzodiazepine at first and, when this was perceived as ineffective, adding a neuroleptic antipsychotic.

Psychotropics are a double-edged sword. Used appropriately, they can reduce distressing symptoms and enhance function. Drugs, however, are increasingly replacing human contact. As we see here, medication side effects in nursing homes can be deleterious. Federal regulations enforced under the Omnibus Budget Reconciliation Act of 1987 have helped reduce the inappropriate use of psychotropics as “chemical restraints.” Still, the emotional distress for patient and caretaker during transitions often leads to inappropriate reliance on psychotropics for predictable adjustment symptoms.

Benzodiazepines have been found to cause sedation, falls, and cognitive clouding and thus should be avoided in older patients. Haloperidol has long been used in psychosis, but its use in older patients is contraindicated because of its side-effect profile. Extrapyramidal symptoms (EPS) are a common side effect of neuroleptics in older persons and are associated with a high incidence of tardive dyskinesia, gait disturbance, akathisia, and cognitive impairment.

Atypical antipsychotics have a more benign side-effect profile (Table 2) and should constitute first-line treatment—but only after human contact, stimulation, and care have been attempted.

Continued treatment: Another setback

A week after starting on haloperidol, Mrs. S fell and fractured her hip. She was transferred to the general hospital, where a surgical repair was performed. Her recovery was slow and difficult. She would not participate in physical therapy and required much coaxing to walk or stand up, often insisting that she could no longer do either. She developed urinary incontinence and became increasingly unable to care for herself. She remained in the hospital for 1 week, then was transferred to a rehabilitation facility.

Dr. Verma’s observations

A causal relationship between Mrs. S’ fall and the haloperidol/oxazepam combination is more than likely. Older persons have diminished pro-prioception, walk on a wider base, and struggle with postural sway. EPS combined with sedation can therefore have disastrous consequences, as this case clearly illustrates. Benzodiazepines, anticholinergics, antihistamines, and the typical neuroleptics are known to impair mobility. Many antidepressants, especially tertiary tricyclics such as amitriptyline and imipramine, may lead to falls by causing orthostatic hypotension secondary to alpha-adrenergic receptor blockade.

Check for pre-treatment orthostatic changes in blood pressure before prescribing psychotropics to older patients. An ECG can also help rule out rhythm abnormalities and assess baseline QTc interval. Agents with the most benign side-effect profiles—atypical antipsychotics, SSRIs, and newer antidepressants such as bupropion and venlafaxine—are recommended for older patients. For Mrs. S, a low dosage of a novel antipsychotic instead of haloperidol and oxazepam would have preserved her physical function and might have greatly reduced her chances of falling.

Follow-up ‘:Not helping’

Mrs. S has been in the nursing home for about 3 1/2 weeks. Staff members consistently report that she “is not helping herself,” is “always weepy,” and “feels her family (has) abandoned her.” She is now taking oxazepam, 15 mg bid, haloperidol, 0.5 mg bid and 1.0 mg at bedtime, and diphenhydramine, 50 mg at bedtime, to help her sleep.

Staff members also report that Mrs. S is “confused and very forgetful … (she) may have Alzheimer’s disease.” Urinary and bowel incontinence are an increasing problem, and she has lost about 15 pounds since she entered the facility. Laboratory readings are normal, but oral intake is poor.

 

 

Could Mrs. S’ functional decline have been avoided? How would you improve her function at this point?

Dr. Verma’s observations

A well-intentioned but ill-conceived drug regimen has compounded problems caused by the prior intervention. As Mrs. S becomes increasingly unable to function—and the staff becomes more frustrated with her deterioration and lack of initiative—more drugs are added. The three agents she is taking all carry a significant risk for sedation, and the anticholinergic effects of both haloperidol and diphenhydramine compound Mrs S’ difficulties by causing delirium and cognitive difficulties.

If this regimen is not modified, Mrs. S likely will stay bed-bound, her cognition will remain impaired or worsen, and her incontinence will continue unchecked. She will require more and more nursing time as her condition deteriorates further. Decubiti, sepsis, and even premature death are all likely sequelae.

In retrospect, an initial intervention with an antidepressant and/or an acetylcholinesterase inhibitor might have prevented such a precipitous decline. It is conceivable that Mrs. S could even have avoided institutional placement. At this point, I would gradually wean her off haloperidol and oxazepam, then aggressively treat her depression, resorting to electroconvulsive therapy if necessary.

In today’s atmosphere of cost containment, care decisions are too often dictated by shortsighted formulary lists, not sound clinical reasoning. In this case, the use of more cost-effective drugs with well-documented higher toxicity ultimately led to excess disability, which in turn required increased effort (and cost) by the treatment team.

Psychosocial interventions can be time-consuming, but they might have prevented Mrs. S’ rapid decline and saved substantial staff time. A higher-functioning patient uses far fewer staff resources, and the added expense of treating a hip fracture exceeds any savings from the use of a lessexpensive medication.

Related resources

  • Salzman C. Psychiatric medications for older adults–the concise guide. New York: Guilford Press, 2001
  • Jacobson SA, Pies RW, Greenblatt DJ. Handbook of geriatric psychopharmacology. Washington DC: American Psychiatric Publishing, 2002.

Drug brand names

  • Amitriptyline • Elavil
  • Bupropion • Wellbutrin
  • Clozapine • Clozaril
  • Donepezil • Aricept
  • Galantamine • Reminyl
  • Haloperidol • Haldol
  • Imipramine • Tofranil
  • Olanzapine • Zyprexa
  • Oxazepam • Serax
  • Quetiapine • Seroquel
  • Risperidone • Risperdal
  • Rivastigmine • Exelon
  • Venlafaxine • Effexor
  • Ziprasidone • Geodon

Disclosure

Dr. Verma reports that he is on the speakers bureau of Eli Lilly and Co. and Abbott Laboratories, serves as a consultant to Eli Lilly and Co., and receives grant support from Eli Lilly and Co. and GlaxoSmithKline.

References

1. Kral VA. Senescent forgetfulness: benign and malignant. Can Med Assoc J 1962;86:257-60.

2. Alexopoulos GS, Meyers BS, Young RC. The vascular depression hypothesis. Arch Gen Psychiatry 1997;54:915-22.

3. Krishnan KR, Hays JC, Blazer DG. MRI defined vascular depression. Am J Psychiatry 1997;154:497-501.

Article PDF
Author and Disclosure Information

Sumer Verma, MD
Lecturer on psychiatry, Harvard Medical School Associate professor of psychiatry, Boston University School of Medicine

Issue
Current Psychiatry - 02(01)
Publications
Topics
Page Number
59-64
Sections
Author and Disclosure Information

Sumer Verma, MD
Lecturer on psychiatry, Harvard Medical School Associate professor of psychiatry, Boston University School of Medicine

Author and Disclosure Information

Sumer Verma, MD
Lecturer on psychiatry, Harvard Medical School Associate professor of psychiatry, Boston University School of Medicine

Article PDF
Article PDF

History: A fading memory

Mrs. S, 85, lives alone in her home of 40 years. Over the past 3 years, she has complained increasingly about headaches, fatigue, and back pain. The cause of these vague physical difficulties has not been determined.

Her daughters say that Mrs. S has become increasingly forgetful. She often does not remember family visits, has difficulty organizing her bank accounts, repeatedly misplaces her pocketbook, and on one occasion became lost on her way to the supermarket. Once fairly social, she has become increasingly isolated.

How would you address Mrs. S’ impaired memory? What medical or psychiatric problems might her forgetfulness indicate?

Dr. Verma’s observations

This case illustrates the fundamentals of geriatric care, the first of which is to preserve—if not enhance—the patient’s function (Box). Forty years ago, Kral1 identified “benign senescent forgetfulness” as a normal aspect of aging. Current research, however, suggests that “senescent forgetfulness” is not always benign. Alexopoulos, Krishnan, and others2,3 have shown that depression manifesting in late life is accompanied by significant white-matter change and substantially increases the risk of developing dementia.

Well past the point of isolated forgetfulness, Mrs. S is exhibiting functional decline and cognitive impairment in multiple domains. The question is, are these symptoms the result of a medical problem such as Alzheimer’s disease or dementia, a psychiatric disorder, or both? The workup and management of these complaints can dramatically affect subsequent outcomes.

Vague medical complaints of unknown cause should not necessarily imply that the problem is psychiatric. The clinician should rule out common medical causes of cognitive decline, including:

  • drug toxicity, especially after anxiolytic and sedative-hypnotic agents have been administered
  • endocrine dysfunction, such as hypothyroidism
  • and CNS neoplasms (Table 1).

Box

10 FUNDAMENTALS OF GERIATRIC CARE

  1. Preserve—if not enhance—the patient’s function
  2. Respect equilibrium; even apparently simple drug interventions can unintentionally upset a precarious adaptive reserve and trigger functional decline
  3. Beware of drug-drug interactions; most older persons are taking multiple medications
  4. Start low and go slow, but do not stop at subtherapeutic dosages
  5. Carefully consider a psychotropic’s side-effect profile
  6. Nondrug interventions are almost always safer than drugs
  7. Drugs do not replace compassion and caring
  8. Two (or more) drugs are not better than one
  9. One drug does not fit all patients
  10. Primum non nocere

A detailed history (still the best diagnostic procedure), a thorough physical evaluation, and routine lab tests can usually help rule out most of these causes. On the other hand, affirmative diagnosis when psychiatric symptoms are evident can minimize testing that can be emotionally, physically, and financially draining to the patient. In Mrs. S’ case, the prominence of the cognitive decline and attendant social withdrawal clearly point to depression or dementia.

The significant overlap between depression and dementia further complicates the diagnosis. Neuropsychological testing can uncover distinguishing factors, but it may help to empirically consider that all late-life depression with cognitive impairment may be secondary to early dementia.

Initiating early drug treatment of dementia with a cholinesterase inhibitor such as donepezil, galantamine, or rivastigmine may slow the trajectory of decline. Vascular risk factors—hypertension and diabetes in particular—also need to be controlled. Low-dose aspirin may help prevent microembolic phenomena.

A selective serotonin reuptake inhibitor (SSRI) can alleviate the depression. If mild paranoia is noted, adding an atypical antipsychotic at a low dosage (olanzapine, 1.25 to 5 mg once daily or risperidone, 0.5 to 1.0 mg/d divided in two doses) may help.

Above all, encourage the patient to remain physically and mentally active. To this end, the clinician should enlist the family and other caregivers to help motivate the patient. Involvement in a day program or similar program may alleviate the patient’s social isolation.

Treatment: New surroundings

Since her initial evaluation 1 year ago, Mrs. S reluctantly has moved into an assisted living facility at her daughters’ insistence. She adjusted well—at least for the first month or so. She then starting calling her daughters at all hours, complaining of being alone and scared. She was taken to a new internist, who prescribed oxazepam, 15 mg bid, for an “anxiety disorder.”

Instead of adjusting to her new surroundings, Mrs. S began to withdraw further. She stayed in her room most days, not even venturing to the dining room for meals. Her personal hygiene deteriorated. According to staff reports, “Mrs. S did not mix with the other residents,” and was becoming “increasingly paranoid.” Her calls to her family had escalated into bitter complaints that people were stealing her belongings.

Table 1

COMMON CAUSES OF DEMENTIA

ReversibleIrreversible
DepressionAlzheimer’s-type dementia
DeliriumVascular dementia
Drug toxicityLess common causes
 Anxiolytics Lewy body dementia
 Sedative-hypnotic drugs Parkinsonian dementia
  Pick’s disease

Mrs. S again visited the internist who, upon hearing that the patient was becoming more paranoid, assumed that she was exhibiting psychotic features. The internist diagnosed Mrs. S as having late-onset Alzheimer’s-type dementia with delusions and added haloperidol, 0.5 mg tid, to her regimen.

 

 

How would you help Mrs. S adjust to her new surroundings? How would your treatment plan differ from that of the internist?Table 2

ANTIPSYCHOTICS: SIDE-EFFECT PROFILES

 Haloperidol 0.75-2 mg/dClozapine 25-100 mg/dRisperidone 0.5-2 mg/dOlanzapine 5-15 mg/dQuetiapine 25-700 mg/d*Ziprasidone 40-160 mg/d*
Anticholinergic effects+/-++----
Sedation+++++++++++
Extrapyramidal symptoms++++-+---
Tardive dyskinesia++---??
Hypotension++++++-+-
QTc prolongation+?---+
Weight gain+++++++++/-
Diabetes mellitus+++++?
* Side effects are probably more severe at higher dosages, but the data are not clear.

Dr. Verma’s observations

The decision to transfer a loved one to a nursing home is difficult for all concerned. I have often seen caregiver “burnout” play a major role in the family’s decision.

After 40 years in her own home, Mrs. S is not likely to adjust readily to living in a “regimented” environment, no matter how comfortable and elegant it may seem. The phenomenon is often called “transfer trauma” and manifests as a sharp decline in function upon moving to a new environment. Most individuals do adapt with time; involving Mrs. S. in a socialization program and insisting on her presence during meals and at other facility events would have hastened her adjustment. Above all, clinicians should be supportive and avoid resorting to medication too soon.

Because Mrs. S’ functional decline was so sharp, however, trying a nondrug therapy would have been easier said than done. Indeed, the internist resorted too quickly to medication, prescribing a short-acting benzodiazepine at first and, when this was perceived as ineffective, adding a neuroleptic antipsychotic.

Psychotropics are a double-edged sword. Used appropriately, they can reduce distressing symptoms and enhance function. Drugs, however, are increasingly replacing human contact. As we see here, medication side effects in nursing homes can be deleterious. Federal regulations enforced under the Omnibus Budget Reconciliation Act of 1987 have helped reduce the inappropriate use of psychotropics as “chemical restraints.” Still, the emotional distress for patient and caretaker during transitions often leads to inappropriate reliance on psychotropics for predictable adjustment symptoms.

Benzodiazepines have been found to cause sedation, falls, and cognitive clouding and thus should be avoided in older patients. Haloperidol has long been used in psychosis, but its use in older patients is contraindicated because of its side-effect profile. Extrapyramidal symptoms (EPS) are a common side effect of neuroleptics in older persons and are associated with a high incidence of tardive dyskinesia, gait disturbance, akathisia, and cognitive impairment.

Atypical antipsychotics have a more benign side-effect profile (Table 2) and should constitute first-line treatment—but only after human contact, stimulation, and care have been attempted.

Continued treatment: Another setback

A week after starting on haloperidol, Mrs. S fell and fractured her hip. She was transferred to the general hospital, where a surgical repair was performed. Her recovery was slow and difficult. She would not participate in physical therapy and required much coaxing to walk or stand up, often insisting that she could no longer do either. She developed urinary incontinence and became increasingly unable to care for herself. She remained in the hospital for 1 week, then was transferred to a rehabilitation facility.

Dr. Verma’s observations

A causal relationship between Mrs. S’ fall and the haloperidol/oxazepam combination is more than likely. Older persons have diminished pro-prioception, walk on a wider base, and struggle with postural sway. EPS combined with sedation can therefore have disastrous consequences, as this case clearly illustrates. Benzodiazepines, anticholinergics, antihistamines, and the typical neuroleptics are known to impair mobility. Many antidepressants, especially tertiary tricyclics such as amitriptyline and imipramine, may lead to falls by causing orthostatic hypotension secondary to alpha-adrenergic receptor blockade.

Check for pre-treatment orthostatic changes in blood pressure before prescribing psychotropics to older patients. An ECG can also help rule out rhythm abnormalities and assess baseline QTc interval. Agents with the most benign side-effect profiles—atypical antipsychotics, SSRIs, and newer antidepressants such as bupropion and venlafaxine—are recommended for older patients. For Mrs. S, a low dosage of a novel antipsychotic instead of haloperidol and oxazepam would have preserved her physical function and might have greatly reduced her chances of falling.

Follow-up ‘:Not helping’

Mrs. S has been in the nursing home for about 3 1/2 weeks. Staff members consistently report that she “is not helping herself,” is “always weepy,” and “feels her family (has) abandoned her.” She is now taking oxazepam, 15 mg bid, haloperidol, 0.5 mg bid and 1.0 mg at bedtime, and diphenhydramine, 50 mg at bedtime, to help her sleep.

Staff members also report that Mrs. S is “confused and very forgetful … (she) may have Alzheimer’s disease.” Urinary and bowel incontinence are an increasing problem, and she has lost about 15 pounds since she entered the facility. Laboratory readings are normal, but oral intake is poor.

 

 

Could Mrs. S’ functional decline have been avoided? How would you improve her function at this point?

Dr. Verma’s observations

A well-intentioned but ill-conceived drug regimen has compounded problems caused by the prior intervention. As Mrs. S becomes increasingly unable to function—and the staff becomes more frustrated with her deterioration and lack of initiative—more drugs are added. The three agents she is taking all carry a significant risk for sedation, and the anticholinergic effects of both haloperidol and diphenhydramine compound Mrs S’ difficulties by causing delirium and cognitive difficulties.

If this regimen is not modified, Mrs. S likely will stay bed-bound, her cognition will remain impaired or worsen, and her incontinence will continue unchecked. She will require more and more nursing time as her condition deteriorates further. Decubiti, sepsis, and even premature death are all likely sequelae.

In retrospect, an initial intervention with an antidepressant and/or an acetylcholinesterase inhibitor might have prevented such a precipitous decline. It is conceivable that Mrs. S could even have avoided institutional placement. At this point, I would gradually wean her off haloperidol and oxazepam, then aggressively treat her depression, resorting to electroconvulsive therapy if necessary.

In today’s atmosphere of cost containment, care decisions are too often dictated by shortsighted formulary lists, not sound clinical reasoning. In this case, the use of more cost-effective drugs with well-documented higher toxicity ultimately led to excess disability, which in turn required increased effort (and cost) by the treatment team.

Psychosocial interventions can be time-consuming, but they might have prevented Mrs. S’ rapid decline and saved substantial staff time. A higher-functioning patient uses far fewer staff resources, and the added expense of treating a hip fracture exceeds any savings from the use of a lessexpensive medication.

Related resources

  • Salzman C. Psychiatric medications for older adults–the concise guide. New York: Guilford Press, 2001
  • Jacobson SA, Pies RW, Greenblatt DJ. Handbook of geriatric psychopharmacology. Washington DC: American Psychiatric Publishing, 2002.

Drug brand names

  • Amitriptyline • Elavil
  • Bupropion • Wellbutrin
  • Clozapine • Clozaril
  • Donepezil • Aricept
  • Galantamine • Reminyl
  • Haloperidol • Haldol
  • Imipramine • Tofranil
  • Olanzapine • Zyprexa
  • Oxazepam • Serax
  • Quetiapine • Seroquel
  • Risperidone • Risperdal
  • Rivastigmine • Exelon
  • Venlafaxine • Effexor
  • Ziprasidone • Geodon

Disclosure

Dr. Verma reports that he is on the speakers bureau of Eli Lilly and Co. and Abbott Laboratories, serves as a consultant to Eli Lilly and Co., and receives grant support from Eli Lilly and Co. and GlaxoSmithKline.

History: A fading memory

Mrs. S, 85, lives alone in her home of 40 years. Over the past 3 years, she has complained increasingly about headaches, fatigue, and back pain. The cause of these vague physical difficulties has not been determined.

Her daughters say that Mrs. S has become increasingly forgetful. She often does not remember family visits, has difficulty organizing her bank accounts, repeatedly misplaces her pocketbook, and on one occasion became lost on her way to the supermarket. Once fairly social, she has become increasingly isolated.

How would you address Mrs. S’ impaired memory? What medical or psychiatric problems might her forgetfulness indicate?

Dr. Verma’s observations

This case illustrates the fundamentals of geriatric care, the first of which is to preserve—if not enhance—the patient’s function (Box). Forty years ago, Kral1 identified “benign senescent forgetfulness” as a normal aspect of aging. Current research, however, suggests that “senescent forgetfulness” is not always benign. Alexopoulos, Krishnan, and others2,3 have shown that depression manifesting in late life is accompanied by significant white-matter change and substantially increases the risk of developing dementia.

Well past the point of isolated forgetfulness, Mrs. S is exhibiting functional decline and cognitive impairment in multiple domains. The question is, are these symptoms the result of a medical problem such as Alzheimer’s disease or dementia, a psychiatric disorder, or both? The workup and management of these complaints can dramatically affect subsequent outcomes.

Vague medical complaints of unknown cause should not necessarily imply that the problem is psychiatric. The clinician should rule out common medical causes of cognitive decline, including:

  • drug toxicity, especially after anxiolytic and sedative-hypnotic agents have been administered
  • endocrine dysfunction, such as hypothyroidism
  • and CNS neoplasms (Table 1).

Box

10 FUNDAMENTALS OF GERIATRIC CARE

  1. Preserve—if not enhance—the patient’s function
  2. Respect equilibrium; even apparently simple drug interventions can unintentionally upset a precarious adaptive reserve and trigger functional decline
  3. Beware of drug-drug interactions; most older persons are taking multiple medications
  4. Start low and go slow, but do not stop at subtherapeutic dosages
  5. Carefully consider a psychotropic’s side-effect profile
  6. Nondrug interventions are almost always safer than drugs
  7. Drugs do not replace compassion and caring
  8. Two (or more) drugs are not better than one
  9. One drug does not fit all patients
  10. Primum non nocere

A detailed history (still the best diagnostic procedure), a thorough physical evaluation, and routine lab tests can usually help rule out most of these causes. On the other hand, affirmative diagnosis when psychiatric symptoms are evident can minimize testing that can be emotionally, physically, and financially draining to the patient. In Mrs. S’ case, the prominence of the cognitive decline and attendant social withdrawal clearly point to depression or dementia.

The significant overlap between depression and dementia further complicates the diagnosis. Neuropsychological testing can uncover distinguishing factors, but it may help to empirically consider that all late-life depression with cognitive impairment may be secondary to early dementia.

Initiating early drug treatment of dementia with a cholinesterase inhibitor such as donepezil, galantamine, or rivastigmine may slow the trajectory of decline. Vascular risk factors—hypertension and diabetes in particular—also need to be controlled. Low-dose aspirin may help prevent microembolic phenomena.

A selective serotonin reuptake inhibitor (SSRI) can alleviate the depression. If mild paranoia is noted, adding an atypical antipsychotic at a low dosage (olanzapine, 1.25 to 5 mg once daily or risperidone, 0.5 to 1.0 mg/d divided in two doses) may help.

Above all, encourage the patient to remain physically and mentally active. To this end, the clinician should enlist the family and other caregivers to help motivate the patient. Involvement in a day program or similar program may alleviate the patient’s social isolation.

Treatment: New surroundings

Since her initial evaluation 1 year ago, Mrs. S reluctantly has moved into an assisted living facility at her daughters’ insistence. She adjusted well—at least for the first month or so. She then starting calling her daughters at all hours, complaining of being alone and scared. She was taken to a new internist, who prescribed oxazepam, 15 mg bid, for an “anxiety disorder.”

Instead of adjusting to her new surroundings, Mrs. S began to withdraw further. She stayed in her room most days, not even venturing to the dining room for meals. Her personal hygiene deteriorated. According to staff reports, “Mrs. S did not mix with the other residents,” and was becoming “increasingly paranoid.” Her calls to her family had escalated into bitter complaints that people were stealing her belongings.

Table 1

COMMON CAUSES OF DEMENTIA

ReversibleIrreversible
DepressionAlzheimer’s-type dementia
DeliriumVascular dementia
Drug toxicityLess common causes
 Anxiolytics Lewy body dementia
 Sedative-hypnotic drugs Parkinsonian dementia
  Pick’s disease

Mrs. S again visited the internist who, upon hearing that the patient was becoming more paranoid, assumed that she was exhibiting psychotic features. The internist diagnosed Mrs. S as having late-onset Alzheimer’s-type dementia with delusions and added haloperidol, 0.5 mg tid, to her regimen.

 

 

How would you help Mrs. S adjust to her new surroundings? How would your treatment plan differ from that of the internist?Table 2

ANTIPSYCHOTICS: SIDE-EFFECT PROFILES

 Haloperidol 0.75-2 mg/dClozapine 25-100 mg/dRisperidone 0.5-2 mg/dOlanzapine 5-15 mg/dQuetiapine 25-700 mg/d*Ziprasidone 40-160 mg/d*
Anticholinergic effects+/-++----
Sedation+++++++++++
Extrapyramidal symptoms++++-+---
Tardive dyskinesia++---??
Hypotension++++++-+-
QTc prolongation+?---+
Weight gain+++++++++/-
Diabetes mellitus+++++?
* Side effects are probably more severe at higher dosages, but the data are not clear.

Dr. Verma’s observations

The decision to transfer a loved one to a nursing home is difficult for all concerned. I have often seen caregiver “burnout” play a major role in the family’s decision.

After 40 years in her own home, Mrs. S is not likely to adjust readily to living in a “regimented” environment, no matter how comfortable and elegant it may seem. The phenomenon is often called “transfer trauma” and manifests as a sharp decline in function upon moving to a new environment. Most individuals do adapt with time; involving Mrs. S. in a socialization program and insisting on her presence during meals and at other facility events would have hastened her adjustment. Above all, clinicians should be supportive and avoid resorting to medication too soon.

Because Mrs. S’ functional decline was so sharp, however, trying a nondrug therapy would have been easier said than done. Indeed, the internist resorted too quickly to medication, prescribing a short-acting benzodiazepine at first and, when this was perceived as ineffective, adding a neuroleptic antipsychotic.

Psychotropics are a double-edged sword. Used appropriately, they can reduce distressing symptoms and enhance function. Drugs, however, are increasingly replacing human contact. As we see here, medication side effects in nursing homes can be deleterious. Federal regulations enforced under the Omnibus Budget Reconciliation Act of 1987 have helped reduce the inappropriate use of psychotropics as “chemical restraints.” Still, the emotional distress for patient and caretaker during transitions often leads to inappropriate reliance on psychotropics for predictable adjustment symptoms.

Benzodiazepines have been found to cause sedation, falls, and cognitive clouding and thus should be avoided in older patients. Haloperidol has long been used in psychosis, but its use in older patients is contraindicated because of its side-effect profile. Extrapyramidal symptoms (EPS) are a common side effect of neuroleptics in older persons and are associated with a high incidence of tardive dyskinesia, gait disturbance, akathisia, and cognitive impairment.

Atypical antipsychotics have a more benign side-effect profile (Table 2) and should constitute first-line treatment—but only after human contact, stimulation, and care have been attempted.

Continued treatment: Another setback

A week after starting on haloperidol, Mrs. S fell and fractured her hip. She was transferred to the general hospital, where a surgical repair was performed. Her recovery was slow and difficult. She would not participate in physical therapy and required much coaxing to walk or stand up, often insisting that she could no longer do either. She developed urinary incontinence and became increasingly unable to care for herself. She remained in the hospital for 1 week, then was transferred to a rehabilitation facility.

Dr. Verma’s observations

A causal relationship between Mrs. S’ fall and the haloperidol/oxazepam combination is more than likely. Older persons have diminished pro-prioception, walk on a wider base, and struggle with postural sway. EPS combined with sedation can therefore have disastrous consequences, as this case clearly illustrates. Benzodiazepines, anticholinergics, antihistamines, and the typical neuroleptics are known to impair mobility. Many antidepressants, especially tertiary tricyclics such as amitriptyline and imipramine, may lead to falls by causing orthostatic hypotension secondary to alpha-adrenergic receptor blockade.

Check for pre-treatment orthostatic changes in blood pressure before prescribing psychotropics to older patients. An ECG can also help rule out rhythm abnormalities and assess baseline QTc interval. Agents with the most benign side-effect profiles—atypical antipsychotics, SSRIs, and newer antidepressants such as bupropion and venlafaxine—are recommended for older patients. For Mrs. S, a low dosage of a novel antipsychotic instead of haloperidol and oxazepam would have preserved her physical function and might have greatly reduced her chances of falling.

Follow-up ‘:Not helping’

Mrs. S has been in the nursing home for about 3 1/2 weeks. Staff members consistently report that she “is not helping herself,” is “always weepy,” and “feels her family (has) abandoned her.” She is now taking oxazepam, 15 mg bid, haloperidol, 0.5 mg bid and 1.0 mg at bedtime, and diphenhydramine, 50 mg at bedtime, to help her sleep.

Staff members also report that Mrs. S is “confused and very forgetful … (she) may have Alzheimer’s disease.” Urinary and bowel incontinence are an increasing problem, and she has lost about 15 pounds since she entered the facility. Laboratory readings are normal, but oral intake is poor.

 

 

Could Mrs. S’ functional decline have been avoided? How would you improve her function at this point?

Dr. Verma’s observations

A well-intentioned but ill-conceived drug regimen has compounded problems caused by the prior intervention. As Mrs. S becomes increasingly unable to function—and the staff becomes more frustrated with her deterioration and lack of initiative—more drugs are added. The three agents she is taking all carry a significant risk for sedation, and the anticholinergic effects of both haloperidol and diphenhydramine compound Mrs S’ difficulties by causing delirium and cognitive difficulties.

If this regimen is not modified, Mrs. S likely will stay bed-bound, her cognition will remain impaired or worsen, and her incontinence will continue unchecked. She will require more and more nursing time as her condition deteriorates further. Decubiti, sepsis, and even premature death are all likely sequelae.

In retrospect, an initial intervention with an antidepressant and/or an acetylcholinesterase inhibitor might have prevented such a precipitous decline. It is conceivable that Mrs. S could even have avoided institutional placement. At this point, I would gradually wean her off haloperidol and oxazepam, then aggressively treat her depression, resorting to electroconvulsive therapy if necessary.

In today’s atmosphere of cost containment, care decisions are too often dictated by shortsighted formulary lists, not sound clinical reasoning. In this case, the use of more cost-effective drugs with well-documented higher toxicity ultimately led to excess disability, which in turn required increased effort (and cost) by the treatment team.

Psychosocial interventions can be time-consuming, but they might have prevented Mrs. S’ rapid decline and saved substantial staff time. A higher-functioning patient uses far fewer staff resources, and the added expense of treating a hip fracture exceeds any savings from the use of a lessexpensive medication.

Related resources

  • Salzman C. Psychiatric medications for older adults–the concise guide. New York: Guilford Press, 2001
  • Jacobson SA, Pies RW, Greenblatt DJ. Handbook of geriatric psychopharmacology. Washington DC: American Psychiatric Publishing, 2002.

Drug brand names

  • Amitriptyline • Elavil
  • Bupropion • Wellbutrin
  • Clozapine • Clozaril
  • Donepezil • Aricept
  • Galantamine • Reminyl
  • Haloperidol • Haldol
  • Imipramine • Tofranil
  • Olanzapine • Zyprexa
  • Oxazepam • Serax
  • Quetiapine • Seroquel
  • Risperidone • Risperdal
  • Rivastigmine • Exelon
  • Venlafaxine • Effexor
  • Ziprasidone • Geodon

Disclosure

Dr. Verma reports that he is on the speakers bureau of Eli Lilly and Co. and Abbott Laboratories, serves as a consultant to Eli Lilly and Co., and receives grant support from Eli Lilly and Co. and GlaxoSmithKline.

References

1. Kral VA. Senescent forgetfulness: benign and malignant. Can Med Assoc J 1962;86:257-60.

2. Alexopoulos GS, Meyers BS, Young RC. The vascular depression hypothesis. Arch Gen Psychiatry 1997;54:915-22.

3. Krishnan KR, Hays JC, Blazer DG. MRI defined vascular depression. Am J Psychiatry 1997;154:497-501.

References

1. Kral VA. Senescent forgetfulness: benign and malignant. Can Med Assoc J 1962;86:257-60.

2. Alexopoulos GS, Meyers BS, Young RC. The vascular depression hypothesis. Arch Gen Psychiatry 1997;54:915-22.

3. Krishnan KR, Hays JC, Blazer DG. MRI defined vascular depression. Am J Psychiatry 1997;154:497-501.

Issue
Current Psychiatry - 02(01)
Issue
Current Psychiatry - 02(01)
Page Number
59-64
Page Number
59-64
Publications
Publications
Topics
Article Type
Display Headline
Treating late-life decline: When more is less
Display Headline
Treating late-life decline: When more is less
Sections
Article Source

PURLs Copyright

Inside the Article

Article PDF Media

Late-life psychosis: It’s efficacy vs. cost in the tug-of-war over treatment

Article Type
Changed
Tue, 12/11/2018 - 15:12
Display Headline
Late-life psychosis: It’s efficacy vs. cost in the tug-of-war over treatment

As psychiatrists, we are being pulled in opposite directions between conflicting goals: to reduce health care costs and to provide our patients with the sophistication and specificity of newly available—and more expensive—biological therapies. In the treatment of late-life psychosis, the evidence is impressive and persuasive that atypical antipsychotic drugs are worth the investment.

Atypical antipsychotics should usually be considered first-line therapy for late-life psychosis, though they cost more than the older neuroleptics. Overall, drugs comprise a minor portion of the cost of treating psychosis but may have a major impact on outcomes.1 With their safer side effect profile, the atypicals have enabled many patients with schizophrenia to “reintegrate” and become contributing members of society. Likewise, the use of atypicals can allow older patients with psychotic symptoms to function longer and more productively in their homes and communities.

Table 1

DIFFERENCES BETWEEN LATE-AND EARLY-ONSET SCHIZOPHRENIA

CharacteristicLate-onset (age 45 and older)Early-onset (before age 45)
GenderMore womenMore men
TypeParanoidVaries
Positive symptomsSevereSevere
Negative symptomsLess severeMore severe
Duration of illnessChronicChronic
CognitionLess impairedMore impaired
NeuroimagingNonspecific changesNonspecific changes
Antipsychotic doseLowerHigher
Mortality ratesHighHigh
Premorbid functionGoodSchizoid traits

Without appropriate treatment of psychosis in younger patients, we know that recurrences and relapses can cause demonstrable brain changes and lead to residual symptoms.2 Older patients have been shown to lose 0.2 “well years” for every year they have psychotic symptoms.3

One illness or many?

Psychotic symptoms can occur in late life for a variety of reasons, and each diagnosis has different implications for patient work-up and treatment. Causes of psychosis in older patients include late-onset schizophrenia, dementia, affective disorder, delusional disorder, and delirium.

Late-life schizophrenia As life expectancy increases, schizophrenia is increasingly being diagnosed in older persons. In a sample of hospitalized patients with schizophrenia, onset of illness occurred after age 50 in 13%, after age 60 in 7%, and after age 70 in 3%.4 The clinical features of schizophrenia may be modified by the concurrent development of dementia. Thus, this reported increase in psychosis after age 50 may be secondary to an increased incidence of dementia, Parkinson’s disease, cerebrovascular events, and neoplasms.

Early definitions of “paraphrenia” and later schizophrenia described delusions and hallucinations that developed without disturbance of affect, with onset in early adult life.5 The occurrence of disorganized behavior and thinking in late life was ascribed to the effects of “senility” or other organic factors.

In 1943, Bleuler6 reported on a series of 126 patients in whom psychosis developed after age 40. In this group, the illness began after age 60 in 4%.

Current diagnostic criteria for schizophrenia do not exclude or categorize individuals on the basis of age. DSMIV does acknowledge a subgroup of patients with “late-onset” schizophrenia after age 45. However, there are important clinical differences in the presentations of the early- and late-onset types (Table 1). For example, even after adjusting for the greater longevity of women, more women than men develop late-onset schizophrenia. Also, compared with the early-onset type, in late-life schizophrenia:

  • Premorbid paranoid or schizoid personality traits appear to be much less prominent.
  • Patients may have had better occupational functioning and are more likely to have been married.
  • Negative symptoms tend to be less severe, although they do contribute to functional decline.

Visual and hearing loss are among the risk factors correlated with late-onset schizophrenia.7 Sensory loss isolates an older person and leads to misinterpretation and misidentification of environmental cues. Other risk factors, in addition to female gender, include cognitive loss, poor social supports, living alone, and alcohol or drug abuse.

Dementia Between 20 and 50% of patients with vascular and mixed dementias exhibit psychotic symptoms.8 Among those with Alzheimer’s dementia, 30% exhibit persecutory delusions.9 Adding to this population are persons with early-onset schizophrenia, who are living longer and can also develop dementia.

While neuroimaging can sometimes aid in diagnosis, abnormalities seen on neuroimaging of patients with psychotic symptoms do not necessarily correlate with cognitive deficits. The clinical significance of these findings is unclear.10,11 Because of differences in therapy, it is important to establish whether the primary diagnosis is dementia or schizophrenia (Table 2).

Affective psychoses Late-life depression and mania are often unrecognized because of atypical presentations. In the elderly, depression may present with withdrawal, mood-incongruent delusions,12 and symptoms that mimic medical illness. Older manic patients may be misidentified as intrusive, hypersexual, or agitated. A high suspicion index, carefully elicited family history, and past psychiatric illness in the patient usually can clarify the diagnosis.

Delusional disorder Suspiciousness and paranoia are common findings in late life, with an estimated prevalence of 4 to 6% in the older population. Patients with delusional disorder show little evidence of cognitive deficits and—unlike those with schizophrenia—continue to maintain a high level of function.13 Delusions are nonbizarre and well systematized, and hallucinations are not a prominent feature.

 

 

Delirium Older persons are particularly vulnerable to developing delirium. Common causes include urinary tract infection, bowel impaction, heart failure, endocrinopathies, and drug toxicity caused by prescribed or over-the-counter drugs. CNS conditions such as subdural hematoma, cerebrovascular accident, slow-growing intracranial tumors, and encephalitis can be associated with the development of hallucinations and delusions without clouding consciousness.

While an agitated, hallucinating patient with clearly evident fluctuations in the sensorium may be readily diagnosed with a delirium, patients who are quiet, withdrawn, and apathetic may be less easy to recognize. Clinicians should therefore be vigilant for both the hyperkinetic and hypokinetic presentations of delirium when assessing older patients.

Work-up

When psychotic symptoms present for the first time in late life, medical causes must be ruled out first. A detailed history, including collateral information from other sources, includes:

  • premorbid function
  • psychiatric history, including history of affective illness
  • family history of schizophrenia, affective illness, and Alzheimer’s disease
  • medical history, including risks for cerebrovascular disease.

The examination should include a complete medical and neurologic evaluation, giving special attention to sensory loss, prescribed and over-the-counter drugs, recent changes in drug regimen, and cognitive screening. Laboratory work-up includes:

  • in all cases: CBC/differential, BUN, TFT, B12/folate, and RPR
  • in selected cases: MRI, HIV screening, toxicology screen, neuropsychological testing
  • in rare cases: lumbar puncture, PET or SPECT imaging.

If the suggested work-up leads to diagnosis of medical illness, begin appropriate treatment. Behavioral symptoms during medical illness, such as with delirium, may require concurrent psychotropic medication until the medical illness is controlled.

If the medical work-up is negative, the clinician then needs to make the appropriate psychiatric diagnosis and institute antipsychotic treatment as indicated.

Table 2

CLINICAL DIFFERENCES BETWEEN DEMENTIA AND SCHIZOPHRENIA

CharacteristicDementiaLate-onset schizophrenia
Incidence50 to 70%1 to 2%
Bizarre delusionsUncommonCommon
HallucinationsUsually visualUsually auditory
History of psychosisRareCommon
MisidentificationCommonLess common
Maintenance treatmentUsually unnecessaryCommon
Antipsychotic dosage25% of dosage for adult schizophrenia50% of dosage for adult schizophrenia

Table 3

USE OF ATYPICAL ANTIPSYCHOTICS IN OLDER PATIENTS

DrugAdministration
ClozapineUse limited by potential for agranulocytosis, need for weekly blood counts
Higher risk for diabetes, hyperglycemia, pancreatitis
Very sedating
“Black box” warning for myocarditis (March 2002)
NOT a first-choice drug for older persons
RisperidoneUsual geriatric dosage 1 to 2 mg/d
Risk of EPS increases significantly with dosages > 2 mg/d
Can cause persistent hyperprolactinemia
OlanzapineUsual geriatric dosage 2.5 to 12.5 mg/d (qd dosing)
Excellent choice as first-line drug in older patients
Anticholinergic side effects ordinarily not a problem in vivo despite receptor profile of M1-M5 antagonism
May be weight-neutral in older patients
QuetiapineGeriatric dosage 25 to 400 mg/d
Very low potential for EPS
Can be a first choice for patients with Lewy body dementia and Parkinson’s disease
Sedation increases with dosages > 200 mg/d
ZiprasidoneGeriatric dosage 40 to 160 mg/d
Significant concern about QT prolongation in older patients, especially women and those with pre-existing cardiac disease, chronic mental illness, hypokalemia, and hypomagnesemia
Not recommended as first-line therapy in elderly (“bold” warning)
Not approved for IM use in United States; restrictions on use in many European countries
EPS: Extrapyramidal symptoms

Prescribing antipsychotics

Prescribing psychoactive medications for older patients is similar to pediatric prescribing. You need to distinguish between behaviors that are “disturbed” as a result of a psychotic process or “disturbing,” for which behavioral interventions may be more appropriate. In any case, pharmacologic and behavioral interventions are used concurrently.

When prescribing for older patients, consider the possible interaction of comorbid medical conditions and altered pharmacokinetics and pharmacodynamics, which increase the potential for drug-drug interactions. Start low and go slow to minimize the potential for adverse side effects and to ensure optimal preservation of function.

In general, atypical antipsychotics are considered first-line therapy, unless there is a compelling reason not to use them in an individual patient. Conventional neuroleptic antipsychotics carry a much higher risk of adverse drug reactions and functional loss without providing substantially greater efficacy.14 Atypical antipsychotics differ not only from the older neuroleptics but also from each other—including their safety profiles for use in older patients (Table 3).

Clozapine is a dibenzodiazepine that is indicated for treatment-resistant schizophrenia. The first atypical antipsychotic, clozapine significantly reduced the incidence of extrapyramidal symptoms (EPS) seen with the neuroleptic antipsychotics. Clozapine also was effective against the “negative” symptoms of schizophrenia such as apathy, anhedonia, and emotional blunting, which severely limited the ability of patients with schizophrenia to function in society.

The limitations of clozapine include its potential to cause agranulocytosis and the requirement for weekly blood tests. It is also fairly sedating. A number of reports have suggested a link between the use of clozapine and the development of type 2 diabetes, hyperglycemia, and pancreatitis.15,16 Earlier this year, the FDA issued a “black box” warning linking the use of clozapine with myocarditis. For all of these reasons, clozapine is not recommended for use in older patients.

 

 

Risperidone is a benzisoxazole derivative indicated for treatment of psychotic disorders. Risperidone’s antipsychotic efficacy is equivalent to that of haloperidol, but it has a much safer side-effect profile. In one multicenter study comparing risperidone and a placebo at single daily dosages of 0.5, 1.0, and 2.0 mg, risperidone was found to be effective in controlling the psychosis and behavioral disturbance associated with dementia. The authors recommended using a dosage of 1.0 mg/d because higher dosages resulted in excessive sedation and EPS.17

Risk of side effects is dose-dependent, especially in the older patient. Risperidone is the only atypical antipsychotic associated with persistent hyperprolactinemia, which may indirectly contribute to increased osteoporosis and atherogenesis.18,19 Orthostatic hypotension, especially with initial dosages greater than 1.5 mg/d and rapid dose escalation (≥ 25% every 24 to 48 hours), may also limit its use in older patients.

Olanzapine is a thiobenzodiazepine derivative indicated for treatment of psychosis and acute bipolar mania. Olanzapine has a receptor profile that is somewhat analogous with that of clozapine. Its antipsychotic efficacy is equivalent to that of risperidone, but it has a more favorable safety profile. Like risperidone, olanzapine is relatively nonsedating, but it is significantly less likely to cause EPS and orthostatic hypotension, and its use is not associated with persistent hyperprolactinemia.

Based on its vitro muscarinic receptor antagonism profile, some clinicians incorrectly assume that olanzapine is highly anticholinergic. In vivo data and clinical experience have not borne out this contention.20,21 Weight gain with olanzapine is relatively infrequent in older persons. A rapid-dissolve preparation can be useful in resistant and noncompliant patients. An IM preparation has been approved by the FDA but has not yet been made available by the manufacturer.

Quetiapine is a dibenzothiazepine derivative indicated for treatment of psychotic disorders. Despite limited data on its efficacy in late-life psychosis, clinical experience would suggest that its antipsychotic efficacy would at least equal that of other drugs in this category.

The need for twice-daily dosing and titration to a therapeutic response can sometimes limit the use of quetiapine. Sedation can be a problem at dosages greater than 200 mg/d. Because quetiapine has a relatively lower potential among the antipsychotics to cause EPS, it may be a first-line choice in patients with Parkinson’s disease and Lewy body dementia.

Ziprasidone is a benzothiazolylpiperazine indicated for treatment of psychosis. Experience with its use in older patients is limited. Ziprasidone should be avoided in patients with significant cardiovascular disease because of its potential to cause QT prolongation and cardiac arrhythmias. Although rare, this cardiac side effect may be life-threatening, and clinicians must be exceptionally vigilant when using ziprasidone in older patients. Risk factors for QT prolongation include older age, female sex, pre-existing cardiac disease, hypokalemia, and hypomagnesemia.

Ziprasidone can be somewhat sedating. An IM preparation is under development but has not been approved for use by the FDA

Neuroleptics and other options

Occasionally a typical neuroleptic may be the most appropriate first-line drug for older patients with psychotic symptoms. For example, a mid-potency neuroleptic such as perphenazine at an IM dose of 2 to 4 mg may be considered when severe agitation and aggression pose a substantial safety risk for the patient or caregiver and require rapid control. Haloperidol, despite its widespread use in both acute and long-term settings, should be used with caution because it has great potential for causing EPS and can immobilize an older patient, resulting in further functional decline. Regardless of which typical neuroleptic is used, switch the patient to an atypical antipsychotic as soon as agitation is under control.

Antidepressants Affective psychoses in older patients may require the addition of an antidepressant to the antipsychotic drug. The selective serotonin reuptake inhibitors fluoxetine and sertraline have proven track records for efficacy and safety and should be considered first-line agents.

Other options Electroconvulsive therapy is safe and effective for older patients with psychotic depression or late-life mania. Late-life mania also may respond well to the anticonvulsant divalproex sodium. Avoid anxiolytics in older patients because of the potential of these agents to cause sedation or disinhibition and their associated risk of falls and confusion. Buspirone in higher dosages (40 to 60 mg/d) can sometimes help manage chronic anxiety states.

Related resource

  • Sadavoy J, Lazarus LW, Jarvik LF, Grossman GT (eds). Comprehensive review of geriatric psychiatry (2nd ed). Washington, DC: American Psychiatric Press, 1996.

Drug brand names

  • Buspirone • Buspar
  • Clozapine • Clozaril
  • Divalproex • Depakote
  • Fluoxetine • Prozac
  • Olanzapine • Zyprexa
  • Perphenazine • Trilafon
  • Quetiapine • Seroquel
  • Risperidone • Risperdal
  • Sertraline • Zoloft
  • Ziprasidone • Geodon

Disclosure

Dr. Snow reports no affiliation or financial arrangement with any of the companies whose products are mentioned in this article.

 

 

Dr. Verma reports that he is on the speakers’ bureau of Eli Lilly and Co. and Abbott Laboratories, serves as a consultant to Eli Lilly and Co., and receives grant support from Eli Lilly and Co. and GlaxoSmithKline.

References

1. Hargreaves WA, Shumway M. Pharmacoeconomics of drug therapy. J Clin Psych 1996;57(suppl 9):66-76.

2. Lieberman J, Chakos M, Wu H, Alvir J, et al. Longitudinal study of brain morphology in first-episode schizophrenia. Biol Psych 2001;49(6):487-99.

3. Patterson TL, Shaw W, Semple SJ, et al. Health-related quality of life in older patients with schizophrenia and other psychoses: relationships among psychosocial and psychiatric factors. Int J Geriatr Psychiatry 1997;12(4):452-61.

4. Harris MJ, Jeste DV. Late-onset schizophrenia: an overview. Schizophr Bull 1988;14:39-55.

5. Kraeplin E. Dementia, praecox, and paraphrenia (1919). Translated by Barclay RM. Huntingdon NY: Krieger, 1971

6. Bleuler E. Late schizophrenic clinical pictures. Fortschr Neurol Psych 1943;15:259-90.

7. Pearlson G, Rabins P. The late-onset psychoses: possible risk factors. Psychiatr Clin North Am 1988;11(1):15-32.

8. Drevets WC, Rubin EH. Psychotic symptoms and the longitudinal course of senile dementia of the Alzheimer’s type. Biol Psychiatry 1989;25:39-48.

9. Wragg R, Jeste DV. Overview of depression and psychosis in Alzheimer’s disease. Am J Psychiatry 1989;146:577-87.

10. Lesser IM. Late-onset psychotic disorder not otherwise specified: clinical and neuroimaging findings. Biol Psychiatry 1992;31:419-23.

11. Lesser IM, Miller BL, Swartz JR, et al. Brain imaging in late-life schizophrenia and related psychoses. Schizophr Bull 1993;19:419-23.

12. Jeste DV, Heaton SC, Paulsen JS, Ercoli L, Harris J, Heaton RK. Clinical and neuropsychological comparison of psychotic depression with nonpsychotic depression and schizophrenia. Am J Psychiatry 1996;153(4):490-6.

13. Burns BJ, Larson DB, Goldstrom ID, et al. Mental disorders among nursing home patients: preliminary findings from the National Nursing Home Survey pretest. Int J Geriatr Psychiatry 1988;3:27-35.

14. Schneider LS, Pollock VE, Lyness SA. A meta-analysis of controlled trials of neuroleptic treatment in dementia. J Am Geriatr Soc 1990;38:553-63.

15. Popli AP, Konicki PE, Jurjus GJ, et al. Clozapine and associated diabetes mellitus. J Clin Psychiatry 1997;58:108-11.

16. Bergemann N, Ehrig C, Diebold K, Mundt C, von Einsiedel R. Asymptomatic pancreatitis associated with clozapine. Pharmacopsychiatry 1999;32(2):78-80.

17. Katz IR, Jeste DV, Mintzer JE, et al. Comparison of risperidone and placebo for psychosis and behavioral disturbances associated with dementia: a randomized, double-blind trial. J Clin Psych 1999;60:107-15.

18. Petty RG. Prolactin and antipsychotic medications: mechanisms of action. Schizophr Res 1999;35(suppl):S67-S73.

19. Dickson RA, Glazer WM. Neuroleptic-induced hyperprolactinemia. Schizophr Res 1999;35(suppl):575-86.

20. Zarate CA, Baldessarini RJ, Siegel AJ, et al:. Risperidone in the elderly: a pharmacoepidemiological study. J Clin Psychiatry 1997;58:311-17.

21. Street JS, Clark WS, Gannon K, et al. Olanzapine treatment of psychotic and behavioral symptoms in patients with Alzheimer’s disease in nursing care facilities: a double-blind, placebo-controlled trial. Arch Gen Psychiatry 2000;57(10):968-76.

Author and Disclosure Information

Reneé E. Snow, MD
McLean Hospital, Belmont, MA Fellow, geriatric psychiatry, Harvard Medical School

Sumer Verma, MD
McLean Hospital, Belmont, MA Lecturer on psychiatry, Harvard Medical School Associate professor of psychiatry Boston University Medical School

Issue
Current Psychiatry - 01(07)
Publications
Topics
Page Number
10-18
Sections
Author and Disclosure Information

Reneé E. Snow, MD
McLean Hospital, Belmont, MA Fellow, geriatric psychiatry, Harvard Medical School

Sumer Verma, MD
McLean Hospital, Belmont, MA Lecturer on psychiatry, Harvard Medical School Associate professor of psychiatry Boston University Medical School

Author and Disclosure Information

Reneé E. Snow, MD
McLean Hospital, Belmont, MA Fellow, geriatric psychiatry, Harvard Medical School

Sumer Verma, MD
McLean Hospital, Belmont, MA Lecturer on psychiatry, Harvard Medical School Associate professor of psychiatry Boston University Medical School

As psychiatrists, we are being pulled in opposite directions between conflicting goals: to reduce health care costs and to provide our patients with the sophistication and specificity of newly available—and more expensive—biological therapies. In the treatment of late-life psychosis, the evidence is impressive and persuasive that atypical antipsychotic drugs are worth the investment.

Atypical antipsychotics should usually be considered first-line therapy for late-life psychosis, though they cost more than the older neuroleptics. Overall, drugs comprise a minor portion of the cost of treating psychosis but may have a major impact on outcomes.1 With their safer side effect profile, the atypicals have enabled many patients with schizophrenia to “reintegrate” and become contributing members of society. Likewise, the use of atypicals can allow older patients with psychotic symptoms to function longer and more productively in their homes and communities.

Table 1

DIFFERENCES BETWEEN LATE-AND EARLY-ONSET SCHIZOPHRENIA

CharacteristicLate-onset (age 45 and older)Early-onset (before age 45)
GenderMore womenMore men
TypeParanoidVaries
Positive symptomsSevereSevere
Negative symptomsLess severeMore severe
Duration of illnessChronicChronic
CognitionLess impairedMore impaired
NeuroimagingNonspecific changesNonspecific changes
Antipsychotic doseLowerHigher
Mortality ratesHighHigh
Premorbid functionGoodSchizoid traits

Without appropriate treatment of psychosis in younger patients, we know that recurrences and relapses can cause demonstrable brain changes and lead to residual symptoms.2 Older patients have been shown to lose 0.2 “well years” for every year they have psychotic symptoms.3

One illness or many?

Psychotic symptoms can occur in late life for a variety of reasons, and each diagnosis has different implications for patient work-up and treatment. Causes of psychosis in older patients include late-onset schizophrenia, dementia, affective disorder, delusional disorder, and delirium.

Late-life schizophrenia As life expectancy increases, schizophrenia is increasingly being diagnosed in older persons. In a sample of hospitalized patients with schizophrenia, onset of illness occurred after age 50 in 13%, after age 60 in 7%, and after age 70 in 3%.4 The clinical features of schizophrenia may be modified by the concurrent development of dementia. Thus, this reported increase in psychosis after age 50 may be secondary to an increased incidence of dementia, Parkinson’s disease, cerebrovascular events, and neoplasms.

Early definitions of “paraphrenia” and later schizophrenia described delusions and hallucinations that developed without disturbance of affect, with onset in early adult life.5 The occurrence of disorganized behavior and thinking in late life was ascribed to the effects of “senility” or other organic factors.

In 1943, Bleuler6 reported on a series of 126 patients in whom psychosis developed after age 40. In this group, the illness began after age 60 in 4%.

Current diagnostic criteria for schizophrenia do not exclude or categorize individuals on the basis of age. DSMIV does acknowledge a subgroup of patients with “late-onset” schizophrenia after age 45. However, there are important clinical differences in the presentations of the early- and late-onset types (Table 1). For example, even after adjusting for the greater longevity of women, more women than men develop late-onset schizophrenia. Also, compared with the early-onset type, in late-life schizophrenia:

  • Premorbid paranoid or schizoid personality traits appear to be much less prominent.
  • Patients may have had better occupational functioning and are more likely to have been married.
  • Negative symptoms tend to be less severe, although they do contribute to functional decline.

Visual and hearing loss are among the risk factors correlated with late-onset schizophrenia.7 Sensory loss isolates an older person and leads to misinterpretation and misidentification of environmental cues. Other risk factors, in addition to female gender, include cognitive loss, poor social supports, living alone, and alcohol or drug abuse.

Dementia Between 20 and 50% of patients with vascular and mixed dementias exhibit psychotic symptoms.8 Among those with Alzheimer’s dementia, 30% exhibit persecutory delusions.9 Adding to this population are persons with early-onset schizophrenia, who are living longer and can also develop dementia.

While neuroimaging can sometimes aid in diagnosis, abnormalities seen on neuroimaging of patients with psychotic symptoms do not necessarily correlate with cognitive deficits. The clinical significance of these findings is unclear.10,11 Because of differences in therapy, it is important to establish whether the primary diagnosis is dementia or schizophrenia (Table 2).

Affective psychoses Late-life depression and mania are often unrecognized because of atypical presentations. In the elderly, depression may present with withdrawal, mood-incongruent delusions,12 and symptoms that mimic medical illness. Older manic patients may be misidentified as intrusive, hypersexual, or agitated. A high suspicion index, carefully elicited family history, and past psychiatric illness in the patient usually can clarify the diagnosis.

Delusional disorder Suspiciousness and paranoia are common findings in late life, with an estimated prevalence of 4 to 6% in the older population. Patients with delusional disorder show little evidence of cognitive deficits and—unlike those with schizophrenia—continue to maintain a high level of function.13 Delusions are nonbizarre and well systematized, and hallucinations are not a prominent feature.

 

 

Delirium Older persons are particularly vulnerable to developing delirium. Common causes include urinary tract infection, bowel impaction, heart failure, endocrinopathies, and drug toxicity caused by prescribed or over-the-counter drugs. CNS conditions such as subdural hematoma, cerebrovascular accident, slow-growing intracranial tumors, and encephalitis can be associated with the development of hallucinations and delusions without clouding consciousness.

While an agitated, hallucinating patient with clearly evident fluctuations in the sensorium may be readily diagnosed with a delirium, patients who are quiet, withdrawn, and apathetic may be less easy to recognize. Clinicians should therefore be vigilant for both the hyperkinetic and hypokinetic presentations of delirium when assessing older patients.

Work-up

When psychotic symptoms present for the first time in late life, medical causes must be ruled out first. A detailed history, including collateral information from other sources, includes:

  • premorbid function
  • psychiatric history, including history of affective illness
  • family history of schizophrenia, affective illness, and Alzheimer’s disease
  • medical history, including risks for cerebrovascular disease.

The examination should include a complete medical and neurologic evaluation, giving special attention to sensory loss, prescribed and over-the-counter drugs, recent changes in drug regimen, and cognitive screening. Laboratory work-up includes:

  • in all cases: CBC/differential, BUN, TFT, B12/folate, and RPR
  • in selected cases: MRI, HIV screening, toxicology screen, neuropsychological testing
  • in rare cases: lumbar puncture, PET or SPECT imaging.

If the suggested work-up leads to diagnosis of medical illness, begin appropriate treatment. Behavioral symptoms during medical illness, such as with delirium, may require concurrent psychotropic medication until the medical illness is controlled.

If the medical work-up is negative, the clinician then needs to make the appropriate psychiatric diagnosis and institute antipsychotic treatment as indicated.

Table 2

CLINICAL DIFFERENCES BETWEEN DEMENTIA AND SCHIZOPHRENIA

CharacteristicDementiaLate-onset schizophrenia
Incidence50 to 70%1 to 2%
Bizarre delusionsUncommonCommon
HallucinationsUsually visualUsually auditory
History of psychosisRareCommon
MisidentificationCommonLess common
Maintenance treatmentUsually unnecessaryCommon
Antipsychotic dosage25% of dosage for adult schizophrenia50% of dosage for adult schizophrenia

Table 3

USE OF ATYPICAL ANTIPSYCHOTICS IN OLDER PATIENTS

DrugAdministration
ClozapineUse limited by potential for agranulocytosis, need for weekly blood counts
Higher risk for diabetes, hyperglycemia, pancreatitis
Very sedating
“Black box” warning for myocarditis (March 2002)
NOT a first-choice drug for older persons
RisperidoneUsual geriatric dosage 1 to 2 mg/d
Risk of EPS increases significantly with dosages > 2 mg/d
Can cause persistent hyperprolactinemia
OlanzapineUsual geriatric dosage 2.5 to 12.5 mg/d (qd dosing)
Excellent choice as first-line drug in older patients
Anticholinergic side effects ordinarily not a problem in vivo despite receptor profile of M1-M5 antagonism
May be weight-neutral in older patients
QuetiapineGeriatric dosage 25 to 400 mg/d
Very low potential for EPS
Can be a first choice for patients with Lewy body dementia and Parkinson’s disease
Sedation increases with dosages > 200 mg/d
ZiprasidoneGeriatric dosage 40 to 160 mg/d
Significant concern about QT prolongation in older patients, especially women and those with pre-existing cardiac disease, chronic mental illness, hypokalemia, and hypomagnesemia
Not recommended as first-line therapy in elderly (“bold” warning)
Not approved for IM use in United States; restrictions on use in many European countries
EPS: Extrapyramidal symptoms

Prescribing antipsychotics

Prescribing psychoactive medications for older patients is similar to pediatric prescribing. You need to distinguish between behaviors that are “disturbed” as a result of a psychotic process or “disturbing,” for which behavioral interventions may be more appropriate. In any case, pharmacologic and behavioral interventions are used concurrently.

When prescribing for older patients, consider the possible interaction of comorbid medical conditions and altered pharmacokinetics and pharmacodynamics, which increase the potential for drug-drug interactions. Start low and go slow to minimize the potential for adverse side effects and to ensure optimal preservation of function.

In general, atypical antipsychotics are considered first-line therapy, unless there is a compelling reason not to use them in an individual patient. Conventional neuroleptic antipsychotics carry a much higher risk of adverse drug reactions and functional loss without providing substantially greater efficacy.14 Atypical antipsychotics differ not only from the older neuroleptics but also from each other—including their safety profiles for use in older patients (Table 3).

Clozapine is a dibenzodiazepine that is indicated for treatment-resistant schizophrenia. The first atypical antipsychotic, clozapine significantly reduced the incidence of extrapyramidal symptoms (EPS) seen with the neuroleptic antipsychotics. Clozapine also was effective against the “negative” symptoms of schizophrenia such as apathy, anhedonia, and emotional blunting, which severely limited the ability of patients with schizophrenia to function in society.

The limitations of clozapine include its potential to cause agranulocytosis and the requirement for weekly blood tests. It is also fairly sedating. A number of reports have suggested a link between the use of clozapine and the development of type 2 diabetes, hyperglycemia, and pancreatitis.15,16 Earlier this year, the FDA issued a “black box” warning linking the use of clozapine with myocarditis. For all of these reasons, clozapine is not recommended for use in older patients.

 

 

Risperidone is a benzisoxazole derivative indicated for treatment of psychotic disorders. Risperidone’s antipsychotic efficacy is equivalent to that of haloperidol, but it has a much safer side-effect profile. In one multicenter study comparing risperidone and a placebo at single daily dosages of 0.5, 1.0, and 2.0 mg, risperidone was found to be effective in controlling the psychosis and behavioral disturbance associated with dementia. The authors recommended using a dosage of 1.0 mg/d because higher dosages resulted in excessive sedation and EPS.17

Risk of side effects is dose-dependent, especially in the older patient. Risperidone is the only atypical antipsychotic associated with persistent hyperprolactinemia, which may indirectly contribute to increased osteoporosis and atherogenesis.18,19 Orthostatic hypotension, especially with initial dosages greater than 1.5 mg/d and rapid dose escalation (≥ 25% every 24 to 48 hours), may also limit its use in older patients.

Olanzapine is a thiobenzodiazepine derivative indicated for treatment of psychosis and acute bipolar mania. Olanzapine has a receptor profile that is somewhat analogous with that of clozapine. Its antipsychotic efficacy is equivalent to that of risperidone, but it has a more favorable safety profile. Like risperidone, olanzapine is relatively nonsedating, but it is significantly less likely to cause EPS and orthostatic hypotension, and its use is not associated with persistent hyperprolactinemia.

Based on its vitro muscarinic receptor antagonism profile, some clinicians incorrectly assume that olanzapine is highly anticholinergic. In vivo data and clinical experience have not borne out this contention.20,21 Weight gain with olanzapine is relatively infrequent in older persons. A rapid-dissolve preparation can be useful in resistant and noncompliant patients. An IM preparation has been approved by the FDA but has not yet been made available by the manufacturer.

Quetiapine is a dibenzothiazepine derivative indicated for treatment of psychotic disorders. Despite limited data on its efficacy in late-life psychosis, clinical experience would suggest that its antipsychotic efficacy would at least equal that of other drugs in this category.

The need for twice-daily dosing and titration to a therapeutic response can sometimes limit the use of quetiapine. Sedation can be a problem at dosages greater than 200 mg/d. Because quetiapine has a relatively lower potential among the antipsychotics to cause EPS, it may be a first-line choice in patients with Parkinson’s disease and Lewy body dementia.

Ziprasidone is a benzothiazolylpiperazine indicated for treatment of psychosis. Experience with its use in older patients is limited. Ziprasidone should be avoided in patients with significant cardiovascular disease because of its potential to cause QT prolongation and cardiac arrhythmias. Although rare, this cardiac side effect may be life-threatening, and clinicians must be exceptionally vigilant when using ziprasidone in older patients. Risk factors for QT prolongation include older age, female sex, pre-existing cardiac disease, hypokalemia, and hypomagnesemia.

Ziprasidone can be somewhat sedating. An IM preparation is under development but has not been approved for use by the FDA

Neuroleptics and other options

Occasionally a typical neuroleptic may be the most appropriate first-line drug for older patients with psychotic symptoms. For example, a mid-potency neuroleptic such as perphenazine at an IM dose of 2 to 4 mg may be considered when severe agitation and aggression pose a substantial safety risk for the patient or caregiver and require rapid control. Haloperidol, despite its widespread use in both acute and long-term settings, should be used with caution because it has great potential for causing EPS and can immobilize an older patient, resulting in further functional decline. Regardless of which typical neuroleptic is used, switch the patient to an atypical antipsychotic as soon as agitation is under control.

Antidepressants Affective psychoses in older patients may require the addition of an antidepressant to the antipsychotic drug. The selective serotonin reuptake inhibitors fluoxetine and sertraline have proven track records for efficacy and safety and should be considered first-line agents.

Other options Electroconvulsive therapy is safe and effective for older patients with psychotic depression or late-life mania. Late-life mania also may respond well to the anticonvulsant divalproex sodium. Avoid anxiolytics in older patients because of the potential of these agents to cause sedation or disinhibition and their associated risk of falls and confusion. Buspirone in higher dosages (40 to 60 mg/d) can sometimes help manage chronic anxiety states.

Related resource

  • Sadavoy J, Lazarus LW, Jarvik LF, Grossman GT (eds). Comprehensive review of geriatric psychiatry (2nd ed). Washington, DC: American Psychiatric Press, 1996.

Drug brand names

  • Buspirone • Buspar
  • Clozapine • Clozaril
  • Divalproex • Depakote
  • Fluoxetine • Prozac
  • Olanzapine • Zyprexa
  • Perphenazine • Trilafon
  • Quetiapine • Seroquel
  • Risperidone • Risperdal
  • Sertraline • Zoloft
  • Ziprasidone • Geodon

Disclosure

Dr. Snow reports no affiliation or financial arrangement with any of the companies whose products are mentioned in this article.

 

 

Dr. Verma reports that he is on the speakers’ bureau of Eli Lilly and Co. and Abbott Laboratories, serves as a consultant to Eli Lilly and Co., and receives grant support from Eli Lilly and Co. and GlaxoSmithKline.

As psychiatrists, we are being pulled in opposite directions between conflicting goals: to reduce health care costs and to provide our patients with the sophistication and specificity of newly available—and more expensive—biological therapies. In the treatment of late-life psychosis, the evidence is impressive and persuasive that atypical antipsychotic drugs are worth the investment.

Atypical antipsychotics should usually be considered first-line therapy for late-life psychosis, though they cost more than the older neuroleptics. Overall, drugs comprise a minor portion of the cost of treating psychosis but may have a major impact on outcomes.1 With their safer side effect profile, the atypicals have enabled many patients with schizophrenia to “reintegrate” and become contributing members of society. Likewise, the use of atypicals can allow older patients with psychotic symptoms to function longer and more productively in their homes and communities.

Table 1

DIFFERENCES BETWEEN LATE-AND EARLY-ONSET SCHIZOPHRENIA

CharacteristicLate-onset (age 45 and older)Early-onset (before age 45)
GenderMore womenMore men
TypeParanoidVaries
Positive symptomsSevereSevere
Negative symptomsLess severeMore severe
Duration of illnessChronicChronic
CognitionLess impairedMore impaired
NeuroimagingNonspecific changesNonspecific changes
Antipsychotic doseLowerHigher
Mortality ratesHighHigh
Premorbid functionGoodSchizoid traits

Without appropriate treatment of psychosis in younger patients, we know that recurrences and relapses can cause demonstrable brain changes and lead to residual symptoms.2 Older patients have been shown to lose 0.2 “well years” for every year they have psychotic symptoms.3

One illness or many?

Psychotic symptoms can occur in late life for a variety of reasons, and each diagnosis has different implications for patient work-up and treatment. Causes of psychosis in older patients include late-onset schizophrenia, dementia, affective disorder, delusional disorder, and delirium.

Late-life schizophrenia As life expectancy increases, schizophrenia is increasingly being diagnosed in older persons. In a sample of hospitalized patients with schizophrenia, onset of illness occurred after age 50 in 13%, after age 60 in 7%, and after age 70 in 3%.4 The clinical features of schizophrenia may be modified by the concurrent development of dementia. Thus, this reported increase in psychosis after age 50 may be secondary to an increased incidence of dementia, Parkinson’s disease, cerebrovascular events, and neoplasms.

Early definitions of “paraphrenia” and later schizophrenia described delusions and hallucinations that developed without disturbance of affect, with onset in early adult life.5 The occurrence of disorganized behavior and thinking in late life was ascribed to the effects of “senility” or other organic factors.

In 1943, Bleuler6 reported on a series of 126 patients in whom psychosis developed after age 40. In this group, the illness began after age 60 in 4%.

Current diagnostic criteria for schizophrenia do not exclude or categorize individuals on the basis of age. DSMIV does acknowledge a subgroup of patients with “late-onset” schizophrenia after age 45. However, there are important clinical differences in the presentations of the early- and late-onset types (Table 1). For example, even after adjusting for the greater longevity of women, more women than men develop late-onset schizophrenia. Also, compared with the early-onset type, in late-life schizophrenia:

  • Premorbid paranoid or schizoid personality traits appear to be much less prominent.
  • Patients may have had better occupational functioning and are more likely to have been married.
  • Negative symptoms tend to be less severe, although they do contribute to functional decline.

Visual and hearing loss are among the risk factors correlated with late-onset schizophrenia.7 Sensory loss isolates an older person and leads to misinterpretation and misidentification of environmental cues. Other risk factors, in addition to female gender, include cognitive loss, poor social supports, living alone, and alcohol or drug abuse.

Dementia Between 20 and 50% of patients with vascular and mixed dementias exhibit psychotic symptoms.8 Among those with Alzheimer’s dementia, 30% exhibit persecutory delusions.9 Adding to this population are persons with early-onset schizophrenia, who are living longer and can also develop dementia.

While neuroimaging can sometimes aid in diagnosis, abnormalities seen on neuroimaging of patients with psychotic symptoms do not necessarily correlate with cognitive deficits. The clinical significance of these findings is unclear.10,11 Because of differences in therapy, it is important to establish whether the primary diagnosis is dementia or schizophrenia (Table 2).

Affective psychoses Late-life depression and mania are often unrecognized because of atypical presentations. In the elderly, depression may present with withdrawal, mood-incongruent delusions,12 and symptoms that mimic medical illness. Older manic patients may be misidentified as intrusive, hypersexual, or agitated. A high suspicion index, carefully elicited family history, and past psychiatric illness in the patient usually can clarify the diagnosis.

Delusional disorder Suspiciousness and paranoia are common findings in late life, with an estimated prevalence of 4 to 6% in the older population. Patients with delusional disorder show little evidence of cognitive deficits and—unlike those with schizophrenia—continue to maintain a high level of function.13 Delusions are nonbizarre and well systematized, and hallucinations are not a prominent feature.

 

 

Delirium Older persons are particularly vulnerable to developing delirium. Common causes include urinary tract infection, bowel impaction, heart failure, endocrinopathies, and drug toxicity caused by prescribed or over-the-counter drugs. CNS conditions such as subdural hematoma, cerebrovascular accident, slow-growing intracranial tumors, and encephalitis can be associated with the development of hallucinations and delusions without clouding consciousness.

While an agitated, hallucinating patient with clearly evident fluctuations in the sensorium may be readily diagnosed with a delirium, patients who are quiet, withdrawn, and apathetic may be less easy to recognize. Clinicians should therefore be vigilant for both the hyperkinetic and hypokinetic presentations of delirium when assessing older patients.

Work-up

When psychotic symptoms present for the first time in late life, medical causes must be ruled out first. A detailed history, including collateral information from other sources, includes:

  • premorbid function
  • psychiatric history, including history of affective illness
  • family history of schizophrenia, affective illness, and Alzheimer’s disease
  • medical history, including risks for cerebrovascular disease.

The examination should include a complete medical and neurologic evaluation, giving special attention to sensory loss, prescribed and over-the-counter drugs, recent changes in drug regimen, and cognitive screening. Laboratory work-up includes:

  • in all cases: CBC/differential, BUN, TFT, B12/folate, and RPR
  • in selected cases: MRI, HIV screening, toxicology screen, neuropsychological testing
  • in rare cases: lumbar puncture, PET or SPECT imaging.

If the suggested work-up leads to diagnosis of medical illness, begin appropriate treatment. Behavioral symptoms during medical illness, such as with delirium, may require concurrent psychotropic medication until the medical illness is controlled.

If the medical work-up is negative, the clinician then needs to make the appropriate psychiatric diagnosis and institute antipsychotic treatment as indicated.

Table 2

CLINICAL DIFFERENCES BETWEEN DEMENTIA AND SCHIZOPHRENIA

CharacteristicDementiaLate-onset schizophrenia
Incidence50 to 70%1 to 2%
Bizarre delusionsUncommonCommon
HallucinationsUsually visualUsually auditory
History of psychosisRareCommon
MisidentificationCommonLess common
Maintenance treatmentUsually unnecessaryCommon
Antipsychotic dosage25% of dosage for adult schizophrenia50% of dosage for adult schizophrenia

Table 3

USE OF ATYPICAL ANTIPSYCHOTICS IN OLDER PATIENTS

DrugAdministration
ClozapineUse limited by potential for agranulocytosis, need for weekly blood counts
Higher risk for diabetes, hyperglycemia, pancreatitis
Very sedating
“Black box” warning for myocarditis (March 2002)
NOT a first-choice drug for older persons
RisperidoneUsual geriatric dosage 1 to 2 mg/d
Risk of EPS increases significantly with dosages > 2 mg/d
Can cause persistent hyperprolactinemia
OlanzapineUsual geriatric dosage 2.5 to 12.5 mg/d (qd dosing)
Excellent choice as first-line drug in older patients
Anticholinergic side effects ordinarily not a problem in vivo despite receptor profile of M1-M5 antagonism
May be weight-neutral in older patients
QuetiapineGeriatric dosage 25 to 400 mg/d
Very low potential for EPS
Can be a first choice for patients with Lewy body dementia and Parkinson’s disease
Sedation increases with dosages > 200 mg/d
ZiprasidoneGeriatric dosage 40 to 160 mg/d
Significant concern about QT prolongation in older patients, especially women and those with pre-existing cardiac disease, chronic mental illness, hypokalemia, and hypomagnesemia
Not recommended as first-line therapy in elderly (“bold” warning)
Not approved for IM use in United States; restrictions on use in many European countries
EPS: Extrapyramidal symptoms

Prescribing antipsychotics

Prescribing psychoactive medications for older patients is similar to pediatric prescribing. You need to distinguish between behaviors that are “disturbed” as a result of a psychotic process or “disturbing,” for which behavioral interventions may be more appropriate. In any case, pharmacologic and behavioral interventions are used concurrently.

When prescribing for older patients, consider the possible interaction of comorbid medical conditions and altered pharmacokinetics and pharmacodynamics, which increase the potential for drug-drug interactions. Start low and go slow to minimize the potential for adverse side effects and to ensure optimal preservation of function.

In general, atypical antipsychotics are considered first-line therapy, unless there is a compelling reason not to use them in an individual patient. Conventional neuroleptic antipsychotics carry a much higher risk of adverse drug reactions and functional loss without providing substantially greater efficacy.14 Atypical antipsychotics differ not only from the older neuroleptics but also from each other—including their safety profiles for use in older patients (Table 3).

Clozapine is a dibenzodiazepine that is indicated for treatment-resistant schizophrenia. The first atypical antipsychotic, clozapine significantly reduced the incidence of extrapyramidal symptoms (EPS) seen with the neuroleptic antipsychotics. Clozapine also was effective against the “negative” symptoms of schizophrenia such as apathy, anhedonia, and emotional blunting, which severely limited the ability of patients with schizophrenia to function in society.

The limitations of clozapine include its potential to cause agranulocytosis and the requirement for weekly blood tests. It is also fairly sedating. A number of reports have suggested a link between the use of clozapine and the development of type 2 diabetes, hyperglycemia, and pancreatitis.15,16 Earlier this year, the FDA issued a “black box” warning linking the use of clozapine with myocarditis. For all of these reasons, clozapine is not recommended for use in older patients.

 

 

Risperidone is a benzisoxazole derivative indicated for treatment of psychotic disorders. Risperidone’s antipsychotic efficacy is equivalent to that of haloperidol, but it has a much safer side-effect profile. In one multicenter study comparing risperidone and a placebo at single daily dosages of 0.5, 1.0, and 2.0 mg, risperidone was found to be effective in controlling the psychosis and behavioral disturbance associated with dementia. The authors recommended using a dosage of 1.0 mg/d because higher dosages resulted in excessive sedation and EPS.17

Risk of side effects is dose-dependent, especially in the older patient. Risperidone is the only atypical antipsychotic associated with persistent hyperprolactinemia, which may indirectly contribute to increased osteoporosis and atherogenesis.18,19 Orthostatic hypotension, especially with initial dosages greater than 1.5 mg/d and rapid dose escalation (≥ 25% every 24 to 48 hours), may also limit its use in older patients.

Olanzapine is a thiobenzodiazepine derivative indicated for treatment of psychosis and acute bipolar mania. Olanzapine has a receptor profile that is somewhat analogous with that of clozapine. Its antipsychotic efficacy is equivalent to that of risperidone, but it has a more favorable safety profile. Like risperidone, olanzapine is relatively nonsedating, but it is significantly less likely to cause EPS and orthostatic hypotension, and its use is not associated with persistent hyperprolactinemia.

Based on its vitro muscarinic receptor antagonism profile, some clinicians incorrectly assume that olanzapine is highly anticholinergic. In vivo data and clinical experience have not borne out this contention.20,21 Weight gain with olanzapine is relatively infrequent in older persons. A rapid-dissolve preparation can be useful in resistant and noncompliant patients. An IM preparation has been approved by the FDA but has not yet been made available by the manufacturer.

Quetiapine is a dibenzothiazepine derivative indicated for treatment of psychotic disorders. Despite limited data on its efficacy in late-life psychosis, clinical experience would suggest that its antipsychotic efficacy would at least equal that of other drugs in this category.

The need for twice-daily dosing and titration to a therapeutic response can sometimes limit the use of quetiapine. Sedation can be a problem at dosages greater than 200 mg/d. Because quetiapine has a relatively lower potential among the antipsychotics to cause EPS, it may be a first-line choice in patients with Parkinson’s disease and Lewy body dementia.

Ziprasidone is a benzothiazolylpiperazine indicated for treatment of psychosis. Experience with its use in older patients is limited. Ziprasidone should be avoided in patients with significant cardiovascular disease because of its potential to cause QT prolongation and cardiac arrhythmias. Although rare, this cardiac side effect may be life-threatening, and clinicians must be exceptionally vigilant when using ziprasidone in older patients. Risk factors for QT prolongation include older age, female sex, pre-existing cardiac disease, hypokalemia, and hypomagnesemia.

Ziprasidone can be somewhat sedating. An IM preparation is under development but has not been approved for use by the FDA

Neuroleptics and other options

Occasionally a typical neuroleptic may be the most appropriate first-line drug for older patients with psychotic symptoms. For example, a mid-potency neuroleptic such as perphenazine at an IM dose of 2 to 4 mg may be considered when severe agitation and aggression pose a substantial safety risk for the patient or caregiver and require rapid control. Haloperidol, despite its widespread use in both acute and long-term settings, should be used with caution because it has great potential for causing EPS and can immobilize an older patient, resulting in further functional decline. Regardless of which typical neuroleptic is used, switch the patient to an atypical antipsychotic as soon as agitation is under control.

Antidepressants Affective psychoses in older patients may require the addition of an antidepressant to the antipsychotic drug. The selective serotonin reuptake inhibitors fluoxetine and sertraline have proven track records for efficacy and safety and should be considered first-line agents.

Other options Electroconvulsive therapy is safe and effective for older patients with psychotic depression or late-life mania. Late-life mania also may respond well to the anticonvulsant divalproex sodium. Avoid anxiolytics in older patients because of the potential of these agents to cause sedation or disinhibition and their associated risk of falls and confusion. Buspirone in higher dosages (40 to 60 mg/d) can sometimes help manage chronic anxiety states.

Related resource

  • Sadavoy J, Lazarus LW, Jarvik LF, Grossman GT (eds). Comprehensive review of geriatric psychiatry (2nd ed). Washington, DC: American Psychiatric Press, 1996.

Drug brand names

  • Buspirone • Buspar
  • Clozapine • Clozaril
  • Divalproex • Depakote
  • Fluoxetine • Prozac
  • Olanzapine • Zyprexa
  • Perphenazine • Trilafon
  • Quetiapine • Seroquel
  • Risperidone • Risperdal
  • Sertraline • Zoloft
  • Ziprasidone • Geodon

Disclosure

Dr. Snow reports no affiliation or financial arrangement with any of the companies whose products are mentioned in this article.

 

 

Dr. Verma reports that he is on the speakers’ bureau of Eli Lilly and Co. and Abbott Laboratories, serves as a consultant to Eli Lilly and Co., and receives grant support from Eli Lilly and Co. and GlaxoSmithKline.

References

1. Hargreaves WA, Shumway M. Pharmacoeconomics of drug therapy. J Clin Psych 1996;57(suppl 9):66-76.

2. Lieberman J, Chakos M, Wu H, Alvir J, et al. Longitudinal study of brain morphology in first-episode schizophrenia. Biol Psych 2001;49(6):487-99.

3. Patterson TL, Shaw W, Semple SJ, et al. Health-related quality of life in older patients with schizophrenia and other psychoses: relationships among psychosocial and psychiatric factors. Int J Geriatr Psychiatry 1997;12(4):452-61.

4. Harris MJ, Jeste DV. Late-onset schizophrenia: an overview. Schizophr Bull 1988;14:39-55.

5. Kraeplin E. Dementia, praecox, and paraphrenia (1919). Translated by Barclay RM. Huntingdon NY: Krieger, 1971

6. Bleuler E. Late schizophrenic clinical pictures. Fortschr Neurol Psych 1943;15:259-90.

7. Pearlson G, Rabins P. The late-onset psychoses: possible risk factors. Psychiatr Clin North Am 1988;11(1):15-32.

8. Drevets WC, Rubin EH. Psychotic symptoms and the longitudinal course of senile dementia of the Alzheimer’s type. Biol Psychiatry 1989;25:39-48.

9. Wragg R, Jeste DV. Overview of depression and psychosis in Alzheimer’s disease. Am J Psychiatry 1989;146:577-87.

10. Lesser IM. Late-onset psychotic disorder not otherwise specified: clinical and neuroimaging findings. Biol Psychiatry 1992;31:419-23.

11. Lesser IM, Miller BL, Swartz JR, et al. Brain imaging in late-life schizophrenia and related psychoses. Schizophr Bull 1993;19:419-23.

12. Jeste DV, Heaton SC, Paulsen JS, Ercoli L, Harris J, Heaton RK. Clinical and neuropsychological comparison of psychotic depression with nonpsychotic depression and schizophrenia. Am J Psychiatry 1996;153(4):490-6.

13. Burns BJ, Larson DB, Goldstrom ID, et al. Mental disorders among nursing home patients: preliminary findings from the National Nursing Home Survey pretest. Int J Geriatr Psychiatry 1988;3:27-35.

14. Schneider LS, Pollock VE, Lyness SA. A meta-analysis of controlled trials of neuroleptic treatment in dementia. J Am Geriatr Soc 1990;38:553-63.

15. Popli AP, Konicki PE, Jurjus GJ, et al. Clozapine and associated diabetes mellitus. J Clin Psychiatry 1997;58:108-11.

16. Bergemann N, Ehrig C, Diebold K, Mundt C, von Einsiedel R. Asymptomatic pancreatitis associated with clozapine. Pharmacopsychiatry 1999;32(2):78-80.

17. Katz IR, Jeste DV, Mintzer JE, et al. Comparison of risperidone and placebo for psychosis and behavioral disturbances associated with dementia: a randomized, double-blind trial. J Clin Psych 1999;60:107-15.

18. Petty RG. Prolactin and antipsychotic medications: mechanisms of action. Schizophr Res 1999;35(suppl):S67-S73.

19. Dickson RA, Glazer WM. Neuroleptic-induced hyperprolactinemia. Schizophr Res 1999;35(suppl):575-86.

20. Zarate CA, Baldessarini RJ, Siegel AJ, et al:. Risperidone in the elderly: a pharmacoepidemiological study. J Clin Psychiatry 1997;58:311-17.

21. Street JS, Clark WS, Gannon K, et al. Olanzapine treatment of psychotic and behavioral symptoms in patients with Alzheimer’s disease in nursing care facilities: a double-blind, placebo-controlled trial. Arch Gen Psychiatry 2000;57(10):968-76.

References

1. Hargreaves WA, Shumway M. Pharmacoeconomics of drug therapy. J Clin Psych 1996;57(suppl 9):66-76.

2. Lieberman J, Chakos M, Wu H, Alvir J, et al. Longitudinal study of brain morphology in first-episode schizophrenia. Biol Psych 2001;49(6):487-99.

3. Patterson TL, Shaw W, Semple SJ, et al. Health-related quality of life in older patients with schizophrenia and other psychoses: relationships among psychosocial and psychiatric factors. Int J Geriatr Psychiatry 1997;12(4):452-61.

4. Harris MJ, Jeste DV. Late-onset schizophrenia: an overview. Schizophr Bull 1988;14:39-55.

5. Kraeplin E. Dementia, praecox, and paraphrenia (1919). Translated by Barclay RM. Huntingdon NY: Krieger, 1971

6. Bleuler E. Late schizophrenic clinical pictures. Fortschr Neurol Psych 1943;15:259-90.

7. Pearlson G, Rabins P. The late-onset psychoses: possible risk factors. Psychiatr Clin North Am 1988;11(1):15-32.

8. Drevets WC, Rubin EH. Psychotic symptoms and the longitudinal course of senile dementia of the Alzheimer’s type. Biol Psychiatry 1989;25:39-48.

9. Wragg R, Jeste DV. Overview of depression and psychosis in Alzheimer’s disease. Am J Psychiatry 1989;146:577-87.

10. Lesser IM. Late-onset psychotic disorder not otherwise specified: clinical and neuroimaging findings. Biol Psychiatry 1992;31:419-23.

11. Lesser IM, Miller BL, Swartz JR, et al. Brain imaging in late-life schizophrenia and related psychoses. Schizophr Bull 1993;19:419-23.

12. Jeste DV, Heaton SC, Paulsen JS, Ercoli L, Harris J, Heaton RK. Clinical and neuropsychological comparison of psychotic depression with nonpsychotic depression and schizophrenia. Am J Psychiatry 1996;153(4):490-6.

13. Burns BJ, Larson DB, Goldstrom ID, et al. Mental disorders among nursing home patients: preliminary findings from the National Nursing Home Survey pretest. Int J Geriatr Psychiatry 1988;3:27-35.

14. Schneider LS, Pollock VE, Lyness SA. A meta-analysis of controlled trials of neuroleptic treatment in dementia. J Am Geriatr Soc 1990;38:553-63.

15. Popli AP, Konicki PE, Jurjus GJ, et al. Clozapine and associated diabetes mellitus. J Clin Psychiatry 1997;58:108-11.

16. Bergemann N, Ehrig C, Diebold K, Mundt C, von Einsiedel R. Asymptomatic pancreatitis associated with clozapine. Pharmacopsychiatry 1999;32(2):78-80.

17. Katz IR, Jeste DV, Mintzer JE, et al. Comparison of risperidone and placebo for psychosis and behavioral disturbances associated with dementia: a randomized, double-blind trial. J Clin Psych 1999;60:107-15.

18. Petty RG. Prolactin and antipsychotic medications: mechanisms of action. Schizophr Res 1999;35(suppl):S67-S73.

19. Dickson RA, Glazer WM. Neuroleptic-induced hyperprolactinemia. Schizophr Res 1999;35(suppl):575-86.

20. Zarate CA, Baldessarini RJ, Siegel AJ, et al:. Risperidone in the elderly: a pharmacoepidemiological study. J Clin Psychiatry 1997;58:311-17.

21. Street JS, Clark WS, Gannon K, et al. Olanzapine treatment of psychotic and behavioral symptoms in patients with Alzheimer’s disease in nursing care facilities: a double-blind, placebo-controlled trial. Arch Gen Psychiatry 2000;57(10):968-76.

Issue
Current Psychiatry - 01(07)
Issue
Current Psychiatry - 01(07)
Page Number
10-18
Page Number
10-18
Publications
Publications
Topics
Article Type
Display Headline
Late-life psychosis: It’s efficacy vs. cost in the tug-of-war over treatment
Display Headline
Late-life psychosis: It’s efficacy vs. cost in the tug-of-war over treatment
Sections
Article Source

PURLs Copyright

Inside the Article

Late-life psychosis: It’s efficacy vs. cost in the tug-of-war over treatment

Article Type
Changed
Tue, 12/11/2018 - 15:12
Display Headline
Late-life psychosis: It’s efficacy vs. cost in the tug-of-war over treatment

As psychiatrists, we are being pulled in opposite directions between conflicting goals: to reduce health care costs and to provide our patients with the sophistication and specificity of newly available—and more expensive—biological therapies. In the treatment of late-life psychosis, the evidence is impressive and persuasive that atypical antipsychotic drugs are worth the investment.

Atypical antipsychotics should usually be considered first-line therapy for late-life psychosis, though they cost more than the older neuroleptics. Overall, drugs comprise a minor portion of the cost of treating psychosis but may have a major impact on outcomes.1 With their safer side effect profile, the atypicals have enabled many patients with schizophrenia to “reintegrate” and become contributing members of society. Likewise, the use of atypicals can allow older patients with psychotic symptoms to function longer and more productively in their homes and communities.

Table 1

DIFFERENCES BETWEEN LATE-AND EARLY-ONSET SCHIZOPHRENIA

CharacteristicLate-onset (age 45 and older)Early-onset (before age 45)
GenderMore womenMore men
TypeParanoidVaries
Positive symptomsSevereSevere
Negative symptomsLess severeMore severe
Duration of illnessChronicChronic
CognitionLess impairedMore impaired
NeuroimagingNonspecific changesNonspecific changes
Antipsychotic doseLowerHigher
Mortality ratesHighHigh
Premorbid functionGoodSchizoid traits

Without appropriate treatment of psychosis in younger patients, we know that recurrences and relapses can cause demonstrable brain changes and lead to residual symptoms.2 Older patients have been shown to lose 0.2 “well years” for every year they have psychotic symptoms.3

One illness or many?

Psychotic symptoms can occur in late life for a variety of reasons, and each diagnosis has different implications for patient work-up and treatment. Causes of psychosis in older patients include late-onset schizophrenia, dementia, affective disorder, delusional disorder, and delirium.

Late-life schizophrenia As life expectancy increases, schizophrenia is increasingly being diagnosed in older persons. In a sample of hospitalized patients with schizophrenia, onset of illness occurred after age 50 in 13%, after age 60 in 7%, and after age 70 in 3%.4 The clinical features of schizophrenia may be modified by the concurrent development of dementia. Thus, this reported increase in psychosis after age 50 may be secondary to an increased incidence of dementia, Parkinson’s disease, cerebrovascular events, and neoplasms.

Early definitions of “paraphrenia” and later schizophrenia described delusions and hallucinations that developed without disturbance of affect, with onset in early adult life.5 The occurrence of disorganized behavior and thinking in late life was ascribed to the effects of “senility” or other organic factors.

In 1943, Bleuler6 reported on a series of 126 patients in whom psychosis developed after age 40. In this group, the illness began after age 60 in 4%.

Current diagnostic criteria for schizophrenia do not exclude or categorize individuals on the basis of age. DSMIV does acknowledge a subgroup of patients with “late-onset” schizophrenia after age 45. However, there are important clinical differences in the presentations of the early- and late-onset types (Table 1). For example, even after adjusting for the greater longevity of women, more women than men develop late-onset schizophrenia. Also, compared with the early-onset type, in late-life schizophrenia:

  • Premorbid paranoid or schizoid personality traits appear to be much less prominent.
  • Patients may have had better occupational functioning and are more likely to have been married.
  • Negative symptoms tend to be less severe, although they do contribute to functional decline.

Visual and hearing loss are among the risk factors correlated with late-onset schizophrenia.7 Sensory loss isolates an older person and leads to misinterpretation and misidentification of environmental cues. Other risk factors, in addition to female gender, include cognitive loss, poor social supports, living alone, and alcohol or drug abuse.

Dementia Between 20 and 50% of patients with vascular and mixed dementias exhibit psychotic symptoms.8 Among those with Alzheimer’s dementia, 30% exhibit persecutory delusions.9 Adding to this population are persons with early-onset schizophrenia, who are living longer and can also develop dementia.

While neuroimaging can sometimes aid in diagnosis, abnormalities seen on neuroimaging of patients with psychotic symptoms do not necessarily correlate with cognitive deficits. The clinical significance of these findings is unclear.10,11 Because of differences in therapy, it is important to establish whether the primary diagnosis is dementia or schizophrenia (Table 2).

Affective psychoses Late-life depression and mania are often unrecognized because of atypical presentations. In the elderly, depression may present with withdrawal, mood-incongruent delusions,12 and symptoms that mimic medical illness. Older manic patients may be misidentified as intrusive, hypersexual, or agitated. A high suspicion index, carefully elicited family history, and past psychiatric illness in the patient usually can clarify the diagnosis.

Delusional disorder Suspiciousness and paranoia are common findings in late life, with an estimated prevalence of 4 to 6% in the older population. Patients with delusional disorder show little evidence of cognitive deficits and—unlike those with schizophrenia—continue to maintain a high level of function.13 Delusions are nonbizarre and well systematized, and hallucinations are not a prominent feature.

 

 

Delirium Older persons are particularly vulnerable to developing delirium. Common causes include urinary tract infection, bowel impaction, heart failure, endocrinopathies, and drug toxicity caused by prescribed or over-the-counter drugs. CNS conditions such as subdural hematoma, cerebrovascular accident, slow-growing intracranial tumors, and encephalitis can be associated with the development of hallucinations and delusions without clouding consciousness.

While an agitated, hallucinating patient with clearly evident fluctuations in the sensorium may be readily diagnosed with a delirium, patients who are quiet, withdrawn, and apathetic may be less easy to recognize. Clinicians should therefore be vigilant for both the hyperkinetic and hypokinetic presentations of delirium when assessing older patients.

Work-up

When psychotic symptoms present for the first time in late life, medical causes must be ruled out first. A detailed history, including collateral information from other sources, includes:

  • premorbid function
  • psychiatric history, including history of affective illness
  • family history of schizophrenia, affective illness, and Alzheimer’s disease
  • medical history, including risks for cerebrovascular disease.

The examination should include a complete medical and neurologic evaluation, giving special attention to sensory loss, prescribed and over-the-counter drugs, recent changes in drug regimen, and cognitive screening. Laboratory work-up includes:

  • in all cases: CBC/differential, BUN, TFT, B12/folate, and RPR
  • in selected cases: MRI, HIV screening, toxicology screen, neuropsychological testing
  • in rare cases: lumbar puncture, PET or SPECT imaging.

If the suggested work-up leads to diagnosis of medical illness, begin appropriate treatment. Behavioral symptoms during medical illness, such as with delirium, may require concurrent psychotropic medication until the medical illness is controlled.

If the medical work-up is negative, the clinician then needs to make the appropriate psychiatric diagnosis and institute antipsychotic treatment as indicated.

Table 2

CLINICAL DIFFERENCES BETWEEN DEMENTIA AND SCHIZOPHRENIA

CharacteristicDementiaLate-onset schizophrenia
Incidence50 to 70%1 to 2%
Bizarre delusionsUncommonCommon
HallucinationsUsually visualUsually auditory
History of psychosisRareCommon
MisidentificationCommonLess common
Maintenance treatmentUsually unnecessaryCommon
Antipsychotic dosage25% of dosage for adult schizophrenia50% of dosage for adult schizophrenia

Table 3

USE OF ATYPICAL ANTIPSYCHOTICS IN OLDER PATIENTS

DrugAdministration
ClozapineUse limited by potential for agranulocytosis, need for weekly blood counts
Higher risk for diabetes, hyperglycemia, pancreatitis
Very sedating
“Black box” warning for myocarditis (March 2002)
NOT a first-choice drug for older persons
RisperidoneUsual geriatric dosage 1 to 2 mg/d
Risk of EPS increases significantly with dosages > 2 mg/d
Can cause persistent hyperprolactinemia
OlanzapineUsual geriatric dosage 2.5 to 12.5 mg/d (qd dosing)
Excellent choice as first-line drug in older patients
Anticholinergic side effects ordinarily not a problem in vivo despite receptor profile of M1-M5 antagonism
May be weight-neutral in older patients
QuetiapineGeriatric dosage 25 to 400 mg/d
Very low potential for EPS
Can be a first choice for patients with Lewy body dementia and Parkinson’s disease
Sedation increases with dosages > 200 mg/d
ZiprasidoneGeriatric dosage 40 to 160 mg/d
Significant concern about QT prolongation in older patients, especially women and those with pre-existing cardiac disease, chronic mental illness, hypokalemia, and hypomagnesemia
Not recommended as first-line therapy in elderly (“bold” warning)
Not approved for IM use in United States; restrictions on use in many European countries
EPS: Extrapyramidal symptoms

Prescribing antipsychotics

Prescribing psychoactive medications for older patients is similar to pediatric prescribing. You need to distinguish between behaviors that are “disturbed” as a result of a psychotic process or “disturbing,” for which behavioral interventions may be more appropriate. In any case, pharmacologic and behavioral interventions are used concurrently.

When prescribing for older patients, consider the possible interaction of comorbid medical conditions and altered pharmacokinetics and pharmacodynamics, which increase the potential for drug-drug interactions. Start low and go slow to minimize the potential for adverse side effects and to ensure optimal preservation of function.

In general, atypical antipsychotics are considered first-line therapy, unless there is a compelling reason not to use them in an individual patient. Conventional neuroleptic antipsychotics carry a much higher risk of adverse drug reactions and functional loss without providing substantially greater efficacy.14 Atypical antipsychotics differ not only from the older neuroleptics but also from each other—including their safety profiles for use in older patients (Table 3).

Clozapine is a dibenzodiazepine that is indicated for treatment-resistant schizophrenia. The first atypical antipsychotic, clozapine significantly reduced the incidence of extrapyramidal symptoms (EPS) seen with the neuroleptic antipsychotics. Clozapine also was effective against the “negative” symptoms of schizophrenia such as apathy, anhedonia, and emotional blunting, which severely limited the ability of patients with schizophrenia to function in society.

The limitations of clozapine include its potential to cause agranulocytosis and the requirement for weekly blood tests. It is also fairly sedating. A number of reports have suggested a link between the use of clozapine and the development of type 2 diabetes, hyperglycemia, and pancreatitis.15,16 Earlier this year, the FDA issued a “black box” warning linking the use of clozapine with myocarditis. For all of these reasons, clozapine is not recommended for use in older patients.

 

 

Risperidone is a benzisoxazole derivative indicated for treatment of psychotic disorders. Risperidone’s antipsychotic efficacy is equivalent to that of haloperidol, but it has a much safer side-effect profile. In one multicenter study comparing risperidone and a placebo at single daily dosages of 0.5, 1.0, and 2.0 mg, risperidone was found to be effective in controlling the psychosis and behavioral disturbance associated with dementia. The authors recommended using a dosage of 1.0 mg/d because higher dosages resulted in excessive sedation and EPS.17

Risk of side effects is dose-dependent, especially in the older patient. Risperidone is the only atypical antipsychotic associated with persistent hyperprolactinemia, which may indirectly contribute to increased osteoporosis and atherogenesis.18,19 Orthostatic hypotension, especially with initial dosages greater than 1.5 mg/d and rapid dose escalation (≥ 25% every 24 to 48 hours), may also limit its use in older patients.

Olanzapine is a thiobenzodiazepine derivative indicated for treatment of psychosis and acute bipolar mania. Olanzapine has a receptor profile that is somewhat analogous with that of clozapine. Its antipsychotic efficacy is equivalent to that of risperidone, but it has a more favorable safety profile. Like risperidone, olanzapine is relatively nonsedating, but it is significantly less likely to cause EPS and orthostatic hypotension, and its use is not associated with persistent hyperprolactinemia.

Based on its vitro muscarinic receptor antagonism profile, some clinicians incorrectly assume that olanzapine is highly anticholinergic. In vivo data and clinical experience have not borne out this contention.20,21 Weight gain with olanzapine is relatively infrequent in older persons. A rapid-dissolve preparation can be useful in resistant and noncompliant patients. An IM preparation has been approved by the FDA but has not yet been made available by the manufacturer.

Quetiapine is a dibenzothiazepine derivative indicated for treatment of psychotic disorders. Despite limited data on its efficacy in late-life psychosis, clinical experience would suggest that its antipsychotic efficacy would at least equal that of other drugs in this category.

The need for twice-daily dosing and titration to a therapeutic response can sometimes limit the use of quetiapine. Sedation can be a problem at dosages greater than 200 mg/d. Because quetiapine has a relatively lower potential among the antipsychotics to cause EPS, it may be a first-line choice in patients with Parkinson’s disease and Lewy body dementia.

Ziprasidone is a benzothiazolylpiperazine indicated for treatment of psychosis. Experience with its use in older patients is limited. Ziprasidone should be avoided in patients with significant cardiovascular disease because of its potential to cause QT prolongation and cardiac arrhythmias. Although rare, this cardiac side effect may be life-threatening, and clinicians must be exceptionally vigilant when using ziprasidone in older patients. Risk factors for QT prolongation include older age, female sex, pre-existing cardiac disease, hypokalemia, and hypomagnesemia.

Ziprasidone can be somewhat sedating. An IM preparation is under development but has not been approved for use by the FDA

Neuroleptics and other options

Occasionally a typical neuroleptic may be the most appropriate first-line drug for older patients with psychotic symptoms. For example, a mid-potency neuroleptic such as perphenazine at an IM dose of 2 to 4 mg may be considered when severe agitation and aggression pose a substantial safety risk for the patient or caregiver and require rapid control. Haloperidol, despite its widespread use in both acute and long-term settings, should be used with caution because it has great potential for causing EPS and can immobilize an older patient, resulting in further functional decline. Regardless of which typical neuroleptic is used, switch the patient to an atypical antipsychotic as soon as agitation is under control.

Antidepressants Affective psychoses in older patients may require the addition of an antidepressant to the antipsychotic drug. The selective serotonin reuptake inhibitors fluoxetine and sertraline have proven track records for efficacy and safety and should be considered first-line agents.

Other options Electroconvulsive therapy is safe and effective for older patients with psychotic depression or late-life mania. Late-life mania also may respond well to the anticonvulsant divalproex sodium. Avoid anxiolytics in older patients because of the potential of these agents to cause sedation or disinhibition and their associated risk of falls and confusion. Buspirone in higher dosages (40 to 60 mg/d) can sometimes help manage chronic anxiety states.

Related resource

  • Sadavoy J, Lazarus LW, Jarvik LF, Grossman GT (eds). Comprehensive review of geriatric psychiatry (2nd ed). Washington, DC: American Psychiatric Press, 1996.

Drug brand names

  • Buspirone • Buspar
  • Clozapine • Clozaril
  • Divalproex • Depakote
  • Fluoxetine • Prozac
  • Olanzapine • Zyprexa
  • Perphenazine • Trilafon
  • Quetiapine • Seroquel
  • Risperidone • Risperdal
  • Sertraline • Zoloft
  • Ziprasidone • Geodon

Disclosure

Dr. Snow reports no affiliation or financial arrangement with any of the companies whose products are mentioned in this article.

 

 

Dr. Verma reports that he is on the speakers’ bureau of Eli Lilly and Co. and Abbott Laboratories, serves as a consultant to Eli Lilly and Co., and receives grant support from Eli Lilly and Co. and GlaxoSmithKline.

References

1. Hargreaves WA, Shumway M. Pharmacoeconomics of drug therapy. J Clin Psych 1996;57(suppl 9):66-76.

2. Lieberman J, Chakos M, Wu H, Alvir J, et al. Longitudinal study of brain morphology in first-episode schizophrenia. Biol Psych 2001;49(6):487-99.

3. Patterson TL, Shaw W, Semple SJ, et al. Health-related quality of life in older patients with schizophrenia and other psychoses: relationships among psychosocial and psychiatric factors. Int J Geriatr Psychiatry 1997;12(4):452-61.

4. Harris MJ, Jeste DV. Late-onset schizophrenia: an overview. Schizophr Bull 1988;14:39-55.

5. Kraeplin E. Dementia, praecox, and paraphrenia (1919). Translated by Barclay RM. Huntingdon NY: Krieger, 1971

6. Bleuler E. Late schizophrenic clinical pictures. Fortschr Neurol Psych 1943;15:259-90.

7. Pearlson G, Rabins P. The late-onset psychoses: possible risk factors. Psychiatr Clin North Am 1988;11(1):15-32.

8. Drevets WC, Rubin EH. Psychotic symptoms and the longitudinal course of senile dementia of the Alzheimer’s type. Biol Psychiatry 1989;25:39-48.

9. Wragg R, Jeste DV. Overview of depression and psychosis in Alzheimer’s disease. Am J Psychiatry 1989;146:577-87.

10. Lesser IM. Late-onset psychotic disorder not otherwise specified: clinical and neuroimaging findings. Biol Psychiatry 1992;31:419-23.

11. Lesser IM, Miller BL, Swartz JR, et al. Brain imaging in late-life schizophrenia and related psychoses. Schizophr Bull 1993;19:419-23.

12. Jeste DV, Heaton SC, Paulsen JS, Ercoli L, Harris J, Heaton RK. Clinical and neuropsychological comparison of psychotic depression with nonpsychotic depression and schizophrenia. Am J Psychiatry 1996;153(4):490-6.

13. Burns BJ, Larson DB, Goldstrom ID, et al. Mental disorders among nursing home patients: preliminary findings from the National Nursing Home Survey pretest. Int J Geriatr Psychiatry 1988;3:27-35.

14. Schneider LS, Pollock VE, Lyness SA. A meta-analysis of controlled trials of neuroleptic treatment in dementia. J Am Geriatr Soc 1990;38:553-63.

15. Popli AP, Konicki PE, Jurjus GJ, et al. Clozapine and associated diabetes mellitus. J Clin Psychiatry 1997;58:108-11.

16. Bergemann N, Ehrig C, Diebold K, Mundt C, von Einsiedel R. Asymptomatic pancreatitis associated with clozapine. Pharmacopsychiatry 1999;32(2):78-80.

17. Katz IR, Jeste DV, Mintzer JE, et al. Comparison of risperidone and placebo for psychosis and behavioral disturbances associated with dementia: a randomized, double-blind trial. J Clin Psych 1999;60:107-15.

18. Petty RG. Prolactin and antipsychotic medications: mechanisms of action. Schizophr Res 1999;35(suppl):S67-S73.

19. Dickson RA, Glazer WM. Neuroleptic-induced hyperprolactinemia. Schizophr Res 1999;35(suppl):575-86.

20. Zarate CA, Baldessarini RJ, Siegel AJ, et al:. Risperidone in the elderly: a pharmacoepidemiological study. J Clin Psychiatry 1997;58:311-17.

21. Street JS, Clark WS, Gannon K, et al. Olanzapine treatment of psychotic and behavioral symptoms in patients with Alzheimer’s disease in nursing care facilities: a double-blind, placebo-controlled trial. Arch Gen Psychiatry 2000;57(10):968-76.

Author and Disclosure Information

Reneé E. Snow, MD
McLean Hospital, Belmont, MA Fellow, geriatric psychiatry, Harvard Medical School

Sumer Verma, MD
McLean Hospital, Belmont, MA Lecturer on psychiatry, Harvard Medical School Associate professor of psychiatry Boston University Medical School

Issue
Current Psychiatry - 01(07)
Publications
Topics
Page Number
10-18
Sections
Author and Disclosure Information

Reneé E. Snow, MD
McLean Hospital, Belmont, MA Fellow, geriatric psychiatry, Harvard Medical School

Sumer Verma, MD
McLean Hospital, Belmont, MA Lecturer on psychiatry, Harvard Medical School Associate professor of psychiatry Boston University Medical School

Author and Disclosure Information

Reneé E. Snow, MD
McLean Hospital, Belmont, MA Fellow, geriatric psychiatry, Harvard Medical School

Sumer Verma, MD
McLean Hospital, Belmont, MA Lecturer on psychiatry, Harvard Medical School Associate professor of psychiatry Boston University Medical School

As psychiatrists, we are being pulled in opposite directions between conflicting goals: to reduce health care costs and to provide our patients with the sophistication and specificity of newly available—and more expensive—biological therapies. In the treatment of late-life psychosis, the evidence is impressive and persuasive that atypical antipsychotic drugs are worth the investment.

Atypical antipsychotics should usually be considered first-line therapy for late-life psychosis, though they cost more than the older neuroleptics. Overall, drugs comprise a minor portion of the cost of treating psychosis but may have a major impact on outcomes.1 With their safer side effect profile, the atypicals have enabled many patients with schizophrenia to “reintegrate” and become contributing members of society. Likewise, the use of atypicals can allow older patients with psychotic symptoms to function longer and more productively in their homes and communities.

Table 1

DIFFERENCES BETWEEN LATE-AND EARLY-ONSET SCHIZOPHRENIA

CharacteristicLate-onset (age 45 and older)Early-onset (before age 45)
GenderMore womenMore men
TypeParanoidVaries
Positive symptomsSevereSevere
Negative symptomsLess severeMore severe
Duration of illnessChronicChronic
CognitionLess impairedMore impaired
NeuroimagingNonspecific changesNonspecific changes
Antipsychotic doseLowerHigher
Mortality ratesHighHigh
Premorbid functionGoodSchizoid traits

Without appropriate treatment of psychosis in younger patients, we know that recurrences and relapses can cause demonstrable brain changes and lead to residual symptoms.2 Older patients have been shown to lose 0.2 “well years” for every year they have psychotic symptoms.3

One illness or many?

Psychotic symptoms can occur in late life for a variety of reasons, and each diagnosis has different implications for patient work-up and treatment. Causes of psychosis in older patients include late-onset schizophrenia, dementia, affective disorder, delusional disorder, and delirium.

Late-life schizophrenia As life expectancy increases, schizophrenia is increasingly being diagnosed in older persons. In a sample of hospitalized patients with schizophrenia, onset of illness occurred after age 50 in 13%, after age 60 in 7%, and after age 70 in 3%.4 The clinical features of schizophrenia may be modified by the concurrent development of dementia. Thus, this reported increase in psychosis after age 50 may be secondary to an increased incidence of dementia, Parkinson’s disease, cerebrovascular events, and neoplasms.

Early definitions of “paraphrenia” and later schizophrenia described delusions and hallucinations that developed without disturbance of affect, with onset in early adult life.5 The occurrence of disorganized behavior and thinking in late life was ascribed to the effects of “senility” or other organic factors.

In 1943, Bleuler6 reported on a series of 126 patients in whom psychosis developed after age 40. In this group, the illness began after age 60 in 4%.

Current diagnostic criteria for schizophrenia do not exclude or categorize individuals on the basis of age. DSMIV does acknowledge a subgroup of patients with “late-onset” schizophrenia after age 45. However, there are important clinical differences in the presentations of the early- and late-onset types (Table 1). For example, even after adjusting for the greater longevity of women, more women than men develop late-onset schizophrenia. Also, compared with the early-onset type, in late-life schizophrenia:

  • Premorbid paranoid or schizoid personality traits appear to be much less prominent.
  • Patients may have had better occupational functioning and are more likely to have been married.
  • Negative symptoms tend to be less severe, although they do contribute to functional decline.

Visual and hearing loss are among the risk factors correlated with late-onset schizophrenia.7 Sensory loss isolates an older person and leads to misinterpretation and misidentification of environmental cues. Other risk factors, in addition to female gender, include cognitive loss, poor social supports, living alone, and alcohol or drug abuse.

Dementia Between 20 and 50% of patients with vascular and mixed dementias exhibit psychotic symptoms.8 Among those with Alzheimer’s dementia, 30% exhibit persecutory delusions.9 Adding to this population are persons with early-onset schizophrenia, who are living longer and can also develop dementia.

While neuroimaging can sometimes aid in diagnosis, abnormalities seen on neuroimaging of patients with psychotic symptoms do not necessarily correlate with cognitive deficits. The clinical significance of these findings is unclear.10,11 Because of differences in therapy, it is important to establish whether the primary diagnosis is dementia or schizophrenia (Table 2).

Affective psychoses Late-life depression and mania are often unrecognized because of atypical presentations. In the elderly, depression may present with withdrawal, mood-incongruent delusions,12 and symptoms that mimic medical illness. Older manic patients may be misidentified as intrusive, hypersexual, or agitated. A high suspicion index, carefully elicited family history, and past psychiatric illness in the patient usually can clarify the diagnosis.

Delusional disorder Suspiciousness and paranoia are common findings in late life, with an estimated prevalence of 4 to 6% in the older population. Patients with delusional disorder show little evidence of cognitive deficits and—unlike those with schizophrenia—continue to maintain a high level of function.13 Delusions are nonbizarre and well systematized, and hallucinations are not a prominent feature.

 

 

Delirium Older persons are particularly vulnerable to developing delirium. Common causes include urinary tract infection, bowel impaction, heart failure, endocrinopathies, and drug toxicity caused by prescribed or over-the-counter drugs. CNS conditions such as subdural hematoma, cerebrovascular accident, slow-growing intracranial tumors, and encephalitis can be associated with the development of hallucinations and delusions without clouding consciousness.

While an agitated, hallucinating patient with clearly evident fluctuations in the sensorium may be readily diagnosed with a delirium, patients who are quiet, withdrawn, and apathetic may be less easy to recognize. Clinicians should therefore be vigilant for both the hyperkinetic and hypokinetic presentations of delirium when assessing older patients.

Work-up

When psychotic symptoms present for the first time in late life, medical causes must be ruled out first. A detailed history, including collateral information from other sources, includes:

  • premorbid function
  • psychiatric history, including history of affective illness
  • family history of schizophrenia, affective illness, and Alzheimer’s disease
  • medical history, including risks for cerebrovascular disease.

The examination should include a complete medical and neurologic evaluation, giving special attention to sensory loss, prescribed and over-the-counter drugs, recent changes in drug regimen, and cognitive screening. Laboratory work-up includes:

  • in all cases: CBC/differential, BUN, TFT, B12/folate, and RPR
  • in selected cases: MRI, HIV screening, toxicology screen, neuropsychological testing
  • in rare cases: lumbar puncture, PET or SPECT imaging.

If the suggested work-up leads to diagnosis of medical illness, begin appropriate treatment. Behavioral symptoms during medical illness, such as with delirium, may require concurrent psychotropic medication until the medical illness is controlled.

If the medical work-up is negative, the clinician then needs to make the appropriate psychiatric diagnosis and institute antipsychotic treatment as indicated.

Table 2

CLINICAL DIFFERENCES BETWEEN DEMENTIA AND SCHIZOPHRENIA

CharacteristicDementiaLate-onset schizophrenia
Incidence50 to 70%1 to 2%
Bizarre delusionsUncommonCommon
HallucinationsUsually visualUsually auditory
History of psychosisRareCommon
MisidentificationCommonLess common
Maintenance treatmentUsually unnecessaryCommon
Antipsychotic dosage25% of dosage for adult schizophrenia50% of dosage for adult schizophrenia

Table 3

USE OF ATYPICAL ANTIPSYCHOTICS IN OLDER PATIENTS

DrugAdministration
ClozapineUse limited by potential for agranulocytosis, need for weekly blood counts
Higher risk for diabetes, hyperglycemia, pancreatitis
Very sedating
“Black box” warning for myocarditis (March 2002)
NOT a first-choice drug for older persons
RisperidoneUsual geriatric dosage 1 to 2 mg/d
Risk of EPS increases significantly with dosages > 2 mg/d
Can cause persistent hyperprolactinemia
OlanzapineUsual geriatric dosage 2.5 to 12.5 mg/d (qd dosing)
Excellent choice as first-line drug in older patients
Anticholinergic side effects ordinarily not a problem in vivo despite receptor profile of M1-M5 antagonism
May be weight-neutral in older patients
QuetiapineGeriatric dosage 25 to 400 mg/d
Very low potential for EPS
Can be a first choice for patients with Lewy body dementia and Parkinson’s disease
Sedation increases with dosages > 200 mg/d
ZiprasidoneGeriatric dosage 40 to 160 mg/d
Significant concern about QT prolongation in older patients, especially women and those with pre-existing cardiac disease, chronic mental illness, hypokalemia, and hypomagnesemia
Not recommended as first-line therapy in elderly (“bold” warning)
Not approved for IM use in United States; restrictions on use in many European countries
EPS: Extrapyramidal symptoms

Prescribing antipsychotics

Prescribing psychoactive medications for older patients is similar to pediatric prescribing. You need to distinguish between behaviors that are “disturbed” as a result of a psychotic process or “disturbing,” for which behavioral interventions may be more appropriate. In any case, pharmacologic and behavioral interventions are used concurrently.

When prescribing for older patients, consider the possible interaction of comorbid medical conditions and altered pharmacokinetics and pharmacodynamics, which increase the potential for drug-drug interactions. Start low and go slow to minimize the potential for adverse side effects and to ensure optimal preservation of function.

In general, atypical antipsychotics are considered first-line therapy, unless there is a compelling reason not to use them in an individual patient. Conventional neuroleptic antipsychotics carry a much higher risk of adverse drug reactions and functional loss without providing substantially greater efficacy.14 Atypical antipsychotics differ not only from the older neuroleptics but also from each other—including their safety profiles for use in older patients (Table 3).

Clozapine is a dibenzodiazepine that is indicated for treatment-resistant schizophrenia. The first atypical antipsychotic, clozapine significantly reduced the incidence of extrapyramidal symptoms (EPS) seen with the neuroleptic antipsychotics. Clozapine also was effective against the “negative” symptoms of schizophrenia such as apathy, anhedonia, and emotional blunting, which severely limited the ability of patients with schizophrenia to function in society.

The limitations of clozapine include its potential to cause agranulocytosis and the requirement for weekly blood tests. It is also fairly sedating. A number of reports have suggested a link between the use of clozapine and the development of type 2 diabetes, hyperglycemia, and pancreatitis.15,16 Earlier this year, the FDA issued a “black box” warning linking the use of clozapine with myocarditis. For all of these reasons, clozapine is not recommended for use in older patients.

 

 

Risperidone is a benzisoxazole derivative indicated for treatment of psychotic disorders. Risperidone’s antipsychotic efficacy is equivalent to that of haloperidol, but it has a much safer side-effect profile. In one multicenter study comparing risperidone and a placebo at single daily dosages of 0.5, 1.0, and 2.0 mg, risperidone was found to be effective in controlling the psychosis and behavioral disturbance associated with dementia. The authors recommended using a dosage of 1.0 mg/d because higher dosages resulted in excessive sedation and EPS.17

Risk of side effects is dose-dependent, especially in the older patient. Risperidone is the only atypical antipsychotic associated with persistent hyperprolactinemia, which may indirectly contribute to increased osteoporosis and atherogenesis.18,19 Orthostatic hypotension, especially with initial dosages greater than 1.5 mg/d and rapid dose escalation (≥ 25% every 24 to 48 hours), may also limit its use in older patients.

Olanzapine is a thiobenzodiazepine derivative indicated for treatment of psychosis and acute bipolar mania. Olanzapine has a receptor profile that is somewhat analogous with that of clozapine. Its antipsychotic efficacy is equivalent to that of risperidone, but it has a more favorable safety profile. Like risperidone, olanzapine is relatively nonsedating, but it is significantly less likely to cause EPS and orthostatic hypotension, and its use is not associated with persistent hyperprolactinemia.

Based on its vitro muscarinic receptor antagonism profile, some clinicians incorrectly assume that olanzapine is highly anticholinergic. In vivo data and clinical experience have not borne out this contention.20,21 Weight gain with olanzapine is relatively infrequent in older persons. A rapid-dissolve preparation can be useful in resistant and noncompliant patients. An IM preparation has been approved by the FDA but has not yet been made available by the manufacturer.

Quetiapine is a dibenzothiazepine derivative indicated for treatment of psychotic disorders. Despite limited data on its efficacy in late-life psychosis, clinical experience would suggest that its antipsychotic efficacy would at least equal that of other drugs in this category.

The need for twice-daily dosing and titration to a therapeutic response can sometimes limit the use of quetiapine. Sedation can be a problem at dosages greater than 200 mg/d. Because quetiapine has a relatively lower potential among the antipsychotics to cause EPS, it may be a first-line choice in patients with Parkinson’s disease and Lewy body dementia.

Ziprasidone is a benzothiazolylpiperazine indicated for treatment of psychosis. Experience with its use in older patients is limited. Ziprasidone should be avoided in patients with significant cardiovascular disease because of its potential to cause QT prolongation and cardiac arrhythmias. Although rare, this cardiac side effect may be life-threatening, and clinicians must be exceptionally vigilant when using ziprasidone in older patients. Risk factors for QT prolongation include older age, female sex, pre-existing cardiac disease, hypokalemia, and hypomagnesemia.

Ziprasidone can be somewhat sedating. An IM preparation is under development but has not been approved for use by the FDA

Neuroleptics and other options

Occasionally a typical neuroleptic may be the most appropriate first-line drug for older patients with psychotic symptoms. For example, a mid-potency neuroleptic such as perphenazine at an IM dose of 2 to 4 mg may be considered when severe agitation and aggression pose a substantial safety risk for the patient or caregiver and require rapid control. Haloperidol, despite its widespread use in both acute and long-term settings, should be used with caution because it has great potential for causing EPS and can immobilize an older patient, resulting in further functional decline. Regardless of which typical neuroleptic is used, switch the patient to an atypical antipsychotic as soon as agitation is under control.

Antidepressants Affective psychoses in older patients may require the addition of an antidepressant to the antipsychotic drug. The selective serotonin reuptake inhibitors fluoxetine and sertraline have proven track records for efficacy and safety and should be considered first-line agents.

Other options Electroconvulsive therapy is safe and effective for older patients with psychotic depression or late-life mania. Late-life mania also may respond well to the anticonvulsant divalproex sodium. Avoid anxiolytics in older patients because of the potential of these agents to cause sedation or disinhibition and their associated risk of falls and confusion. Buspirone in higher dosages (40 to 60 mg/d) can sometimes help manage chronic anxiety states.

Related resource

  • Sadavoy J, Lazarus LW, Jarvik LF, Grossman GT (eds). Comprehensive review of geriatric psychiatry (2nd ed). Washington, DC: American Psychiatric Press, 1996.

Drug brand names

  • Buspirone • Buspar
  • Clozapine • Clozaril
  • Divalproex • Depakote
  • Fluoxetine • Prozac
  • Olanzapine • Zyprexa
  • Perphenazine • Trilafon
  • Quetiapine • Seroquel
  • Risperidone • Risperdal
  • Sertraline • Zoloft
  • Ziprasidone • Geodon

Disclosure

Dr. Snow reports no affiliation or financial arrangement with any of the companies whose products are mentioned in this article.

 

 

Dr. Verma reports that he is on the speakers’ bureau of Eli Lilly and Co. and Abbott Laboratories, serves as a consultant to Eli Lilly and Co., and receives grant support from Eli Lilly and Co. and GlaxoSmithKline.

As psychiatrists, we are being pulled in opposite directions between conflicting goals: to reduce health care costs and to provide our patients with the sophistication and specificity of newly available—and more expensive—biological therapies. In the treatment of late-life psychosis, the evidence is impressive and persuasive that atypical antipsychotic drugs are worth the investment.

Atypical antipsychotics should usually be considered first-line therapy for late-life psychosis, though they cost more than the older neuroleptics. Overall, drugs comprise a minor portion of the cost of treating psychosis but may have a major impact on outcomes.1 With their safer side effect profile, the atypicals have enabled many patients with schizophrenia to “reintegrate” and become contributing members of society. Likewise, the use of atypicals can allow older patients with psychotic symptoms to function longer and more productively in their homes and communities.

Table 1

DIFFERENCES BETWEEN LATE-AND EARLY-ONSET SCHIZOPHRENIA

CharacteristicLate-onset (age 45 and older)Early-onset (before age 45)
GenderMore womenMore men
TypeParanoidVaries
Positive symptomsSevereSevere
Negative symptomsLess severeMore severe
Duration of illnessChronicChronic
CognitionLess impairedMore impaired
NeuroimagingNonspecific changesNonspecific changes
Antipsychotic doseLowerHigher
Mortality ratesHighHigh
Premorbid functionGoodSchizoid traits

Without appropriate treatment of psychosis in younger patients, we know that recurrences and relapses can cause demonstrable brain changes and lead to residual symptoms.2 Older patients have been shown to lose 0.2 “well years” for every year they have psychotic symptoms.3

One illness or many?

Psychotic symptoms can occur in late life for a variety of reasons, and each diagnosis has different implications for patient work-up and treatment. Causes of psychosis in older patients include late-onset schizophrenia, dementia, affective disorder, delusional disorder, and delirium.

Late-life schizophrenia As life expectancy increases, schizophrenia is increasingly being diagnosed in older persons. In a sample of hospitalized patients with schizophrenia, onset of illness occurred after age 50 in 13%, after age 60 in 7%, and after age 70 in 3%.4 The clinical features of schizophrenia may be modified by the concurrent development of dementia. Thus, this reported increase in psychosis after age 50 may be secondary to an increased incidence of dementia, Parkinson’s disease, cerebrovascular events, and neoplasms.

Early definitions of “paraphrenia” and later schizophrenia described delusions and hallucinations that developed without disturbance of affect, with onset in early adult life.5 The occurrence of disorganized behavior and thinking in late life was ascribed to the effects of “senility” or other organic factors.

In 1943, Bleuler6 reported on a series of 126 patients in whom psychosis developed after age 40. In this group, the illness began after age 60 in 4%.

Current diagnostic criteria for schizophrenia do not exclude or categorize individuals on the basis of age. DSMIV does acknowledge a subgroup of patients with “late-onset” schizophrenia after age 45. However, there are important clinical differences in the presentations of the early- and late-onset types (Table 1). For example, even after adjusting for the greater longevity of women, more women than men develop late-onset schizophrenia. Also, compared with the early-onset type, in late-life schizophrenia:

  • Premorbid paranoid or schizoid personality traits appear to be much less prominent.
  • Patients may have had better occupational functioning and are more likely to have been married.
  • Negative symptoms tend to be less severe, although they do contribute to functional decline.

Visual and hearing loss are among the risk factors correlated with late-onset schizophrenia.7 Sensory loss isolates an older person and leads to misinterpretation and misidentification of environmental cues. Other risk factors, in addition to female gender, include cognitive loss, poor social supports, living alone, and alcohol or drug abuse.

Dementia Between 20 and 50% of patients with vascular and mixed dementias exhibit psychotic symptoms.8 Among those with Alzheimer’s dementia, 30% exhibit persecutory delusions.9 Adding to this population are persons with early-onset schizophrenia, who are living longer and can also develop dementia.

While neuroimaging can sometimes aid in diagnosis, abnormalities seen on neuroimaging of patients with psychotic symptoms do not necessarily correlate with cognitive deficits. The clinical significance of these findings is unclear.10,11 Because of differences in therapy, it is important to establish whether the primary diagnosis is dementia or schizophrenia (Table 2).

Affective psychoses Late-life depression and mania are often unrecognized because of atypical presentations. In the elderly, depression may present with withdrawal, mood-incongruent delusions,12 and symptoms that mimic medical illness. Older manic patients may be misidentified as intrusive, hypersexual, or agitated. A high suspicion index, carefully elicited family history, and past psychiatric illness in the patient usually can clarify the diagnosis.

Delusional disorder Suspiciousness and paranoia are common findings in late life, with an estimated prevalence of 4 to 6% in the older population. Patients with delusional disorder show little evidence of cognitive deficits and—unlike those with schizophrenia—continue to maintain a high level of function.13 Delusions are nonbizarre and well systematized, and hallucinations are not a prominent feature.

 

 

Delirium Older persons are particularly vulnerable to developing delirium. Common causes include urinary tract infection, bowel impaction, heart failure, endocrinopathies, and drug toxicity caused by prescribed or over-the-counter drugs. CNS conditions such as subdural hematoma, cerebrovascular accident, slow-growing intracranial tumors, and encephalitis can be associated with the development of hallucinations and delusions without clouding consciousness.

While an agitated, hallucinating patient with clearly evident fluctuations in the sensorium may be readily diagnosed with a delirium, patients who are quiet, withdrawn, and apathetic may be less easy to recognize. Clinicians should therefore be vigilant for both the hyperkinetic and hypokinetic presentations of delirium when assessing older patients.

Work-up

When psychotic symptoms present for the first time in late life, medical causes must be ruled out first. A detailed history, including collateral information from other sources, includes:

  • premorbid function
  • psychiatric history, including history of affective illness
  • family history of schizophrenia, affective illness, and Alzheimer’s disease
  • medical history, including risks for cerebrovascular disease.

The examination should include a complete medical and neurologic evaluation, giving special attention to sensory loss, prescribed and over-the-counter drugs, recent changes in drug regimen, and cognitive screening. Laboratory work-up includes:

  • in all cases: CBC/differential, BUN, TFT, B12/folate, and RPR
  • in selected cases: MRI, HIV screening, toxicology screen, neuropsychological testing
  • in rare cases: lumbar puncture, PET or SPECT imaging.

If the suggested work-up leads to diagnosis of medical illness, begin appropriate treatment. Behavioral symptoms during medical illness, such as with delirium, may require concurrent psychotropic medication until the medical illness is controlled.

If the medical work-up is negative, the clinician then needs to make the appropriate psychiatric diagnosis and institute antipsychotic treatment as indicated.

Table 2

CLINICAL DIFFERENCES BETWEEN DEMENTIA AND SCHIZOPHRENIA

CharacteristicDementiaLate-onset schizophrenia
Incidence50 to 70%1 to 2%
Bizarre delusionsUncommonCommon
HallucinationsUsually visualUsually auditory
History of psychosisRareCommon
MisidentificationCommonLess common
Maintenance treatmentUsually unnecessaryCommon
Antipsychotic dosage25% of dosage for adult schizophrenia50% of dosage for adult schizophrenia

Table 3

USE OF ATYPICAL ANTIPSYCHOTICS IN OLDER PATIENTS

DrugAdministration
ClozapineUse limited by potential for agranulocytosis, need for weekly blood counts
Higher risk for diabetes, hyperglycemia, pancreatitis
Very sedating
“Black box” warning for myocarditis (March 2002)
NOT a first-choice drug for older persons
RisperidoneUsual geriatric dosage 1 to 2 mg/d
Risk of EPS increases significantly with dosages > 2 mg/d
Can cause persistent hyperprolactinemia
OlanzapineUsual geriatric dosage 2.5 to 12.5 mg/d (qd dosing)
Excellent choice as first-line drug in older patients
Anticholinergic side effects ordinarily not a problem in vivo despite receptor profile of M1-M5 antagonism
May be weight-neutral in older patients
QuetiapineGeriatric dosage 25 to 400 mg/d
Very low potential for EPS
Can be a first choice for patients with Lewy body dementia and Parkinson’s disease
Sedation increases with dosages > 200 mg/d
ZiprasidoneGeriatric dosage 40 to 160 mg/d
Significant concern about QT prolongation in older patients, especially women and those with pre-existing cardiac disease, chronic mental illness, hypokalemia, and hypomagnesemia
Not recommended as first-line therapy in elderly (“bold” warning)
Not approved for IM use in United States; restrictions on use in many European countries
EPS: Extrapyramidal symptoms

Prescribing antipsychotics

Prescribing psychoactive medications for older patients is similar to pediatric prescribing. You need to distinguish between behaviors that are “disturbed” as a result of a psychotic process or “disturbing,” for which behavioral interventions may be more appropriate. In any case, pharmacologic and behavioral interventions are used concurrently.

When prescribing for older patients, consider the possible interaction of comorbid medical conditions and altered pharmacokinetics and pharmacodynamics, which increase the potential for drug-drug interactions. Start low and go slow to minimize the potential for adverse side effects and to ensure optimal preservation of function.

In general, atypical antipsychotics are considered first-line therapy, unless there is a compelling reason not to use them in an individual patient. Conventional neuroleptic antipsychotics carry a much higher risk of adverse drug reactions and functional loss without providing substantially greater efficacy.14 Atypical antipsychotics differ not only from the older neuroleptics but also from each other—including their safety profiles for use in older patients (Table 3).

Clozapine is a dibenzodiazepine that is indicated for treatment-resistant schizophrenia. The first atypical antipsychotic, clozapine significantly reduced the incidence of extrapyramidal symptoms (EPS) seen with the neuroleptic antipsychotics. Clozapine also was effective against the “negative” symptoms of schizophrenia such as apathy, anhedonia, and emotional blunting, which severely limited the ability of patients with schizophrenia to function in society.

The limitations of clozapine include its potential to cause agranulocytosis and the requirement for weekly blood tests. It is also fairly sedating. A number of reports have suggested a link between the use of clozapine and the development of type 2 diabetes, hyperglycemia, and pancreatitis.15,16 Earlier this year, the FDA issued a “black box” warning linking the use of clozapine with myocarditis. For all of these reasons, clozapine is not recommended for use in older patients.

 

 

Risperidone is a benzisoxazole derivative indicated for treatment of psychotic disorders. Risperidone’s antipsychotic efficacy is equivalent to that of haloperidol, but it has a much safer side-effect profile. In one multicenter study comparing risperidone and a placebo at single daily dosages of 0.5, 1.0, and 2.0 mg, risperidone was found to be effective in controlling the psychosis and behavioral disturbance associated with dementia. The authors recommended using a dosage of 1.0 mg/d because higher dosages resulted in excessive sedation and EPS.17

Risk of side effects is dose-dependent, especially in the older patient. Risperidone is the only atypical antipsychotic associated with persistent hyperprolactinemia, which may indirectly contribute to increased osteoporosis and atherogenesis.18,19 Orthostatic hypotension, especially with initial dosages greater than 1.5 mg/d and rapid dose escalation (≥ 25% every 24 to 48 hours), may also limit its use in older patients.

Olanzapine is a thiobenzodiazepine derivative indicated for treatment of psychosis and acute bipolar mania. Olanzapine has a receptor profile that is somewhat analogous with that of clozapine. Its antipsychotic efficacy is equivalent to that of risperidone, but it has a more favorable safety profile. Like risperidone, olanzapine is relatively nonsedating, but it is significantly less likely to cause EPS and orthostatic hypotension, and its use is not associated with persistent hyperprolactinemia.

Based on its vitro muscarinic receptor antagonism profile, some clinicians incorrectly assume that olanzapine is highly anticholinergic. In vivo data and clinical experience have not borne out this contention.20,21 Weight gain with olanzapine is relatively infrequent in older persons. A rapid-dissolve preparation can be useful in resistant and noncompliant patients. An IM preparation has been approved by the FDA but has not yet been made available by the manufacturer.

Quetiapine is a dibenzothiazepine derivative indicated for treatment of psychotic disorders. Despite limited data on its efficacy in late-life psychosis, clinical experience would suggest that its antipsychotic efficacy would at least equal that of other drugs in this category.

The need for twice-daily dosing and titration to a therapeutic response can sometimes limit the use of quetiapine. Sedation can be a problem at dosages greater than 200 mg/d. Because quetiapine has a relatively lower potential among the antipsychotics to cause EPS, it may be a first-line choice in patients with Parkinson’s disease and Lewy body dementia.

Ziprasidone is a benzothiazolylpiperazine indicated for treatment of psychosis. Experience with its use in older patients is limited. Ziprasidone should be avoided in patients with significant cardiovascular disease because of its potential to cause QT prolongation and cardiac arrhythmias. Although rare, this cardiac side effect may be life-threatening, and clinicians must be exceptionally vigilant when using ziprasidone in older patients. Risk factors for QT prolongation include older age, female sex, pre-existing cardiac disease, hypokalemia, and hypomagnesemia.

Ziprasidone can be somewhat sedating. An IM preparation is under development but has not been approved for use by the FDA

Neuroleptics and other options

Occasionally a typical neuroleptic may be the most appropriate first-line drug for older patients with psychotic symptoms. For example, a mid-potency neuroleptic such as perphenazine at an IM dose of 2 to 4 mg may be considered when severe agitation and aggression pose a substantial safety risk for the patient or caregiver and require rapid control. Haloperidol, despite its widespread use in both acute and long-term settings, should be used with caution because it has great potential for causing EPS and can immobilize an older patient, resulting in further functional decline. Regardless of which typical neuroleptic is used, switch the patient to an atypical antipsychotic as soon as agitation is under control.

Antidepressants Affective psychoses in older patients may require the addition of an antidepressant to the antipsychotic drug. The selective serotonin reuptake inhibitors fluoxetine and sertraline have proven track records for efficacy and safety and should be considered first-line agents.

Other options Electroconvulsive therapy is safe and effective for older patients with psychotic depression or late-life mania. Late-life mania also may respond well to the anticonvulsant divalproex sodium. Avoid anxiolytics in older patients because of the potential of these agents to cause sedation or disinhibition and their associated risk of falls and confusion. Buspirone in higher dosages (40 to 60 mg/d) can sometimes help manage chronic anxiety states.

Related resource

  • Sadavoy J, Lazarus LW, Jarvik LF, Grossman GT (eds). Comprehensive review of geriatric psychiatry (2nd ed). Washington, DC: American Psychiatric Press, 1996.

Drug brand names

  • Buspirone • Buspar
  • Clozapine • Clozaril
  • Divalproex • Depakote
  • Fluoxetine • Prozac
  • Olanzapine • Zyprexa
  • Perphenazine • Trilafon
  • Quetiapine • Seroquel
  • Risperidone • Risperdal
  • Sertraline • Zoloft
  • Ziprasidone • Geodon

Disclosure

Dr. Snow reports no affiliation or financial arrangement with any of the companies whose products are mentioned in this article.

 

 

Dr. Verma reports that he is on the speakers’ bureau of Eli Lilly and Co. and Abbott Laboratories, serves as a consultant to Eli Lilly and Co., and receives grant support from Eli Lilly and Co. and GlaxoSmithKline.

References

1. Hargreaves WA, Shumway M. Pharmacoeconomics of drug therapy. J Clin Psych 1996;57(suppl 9):66-76.

2. Lieberman J, Chakos M, Wu H, Alvir J, et al. Longitudinal study of brain morphology in first-episode schizophrenia. Biol Psych 2001;49(6):487-99.

3. Patterson TL, Shaw W, Semple SJ, et al. Health-related quality of life in older patients with schizophrenia and other psychoses: relationships among psychosocial and psychiatric factors. Int J Geriatr Psychiatry 1997;12(4):452-61.

4. Harris MJ, Jeste DV. Late-onset schizophrenia: an overview. Schizophr Bull 1988;14:39-55.

5. Kraeplin E. Dementia, praecox, and paraphrenia (1919). Translated by Barclay RM. Huntingdon NY: Krieger, 1971

6. Bleuler E. Late schizophrenic clinical pictures. Fortschr Neurol Psych 1943;15:259-90.

7. Pearlson G, Rabins P. The late-onset psychoses: possible risk factors. Psychiatr Clin North Am 1988;11(1):15-32.

8. Drevets WC, Rubin EH. Psychotic symptoms and the longitudinal course of senile dementia of the Alzheimer’s type. Biol Psychiatry 1989;25:39-48.

9. Wragg R, Jeste DV. Overview of depression and psychosis in Alzheimer’s disease. Am J Psychiatry 1989;146:577-87.

10. Lesser IM. Late-onset psychotic disorder not otherwise specified: clinical and neuroimaging findings. Biol Psychiatry 1992;31:419-23.

11. Lesser IM, Miller BL, Swartz JR, et al. Brain imaging in late-life schizophrenia and related psychoses. Schizophr Bull 1993;19:419-23.

12. Jeste DV, Heaton SC, Paulsen JS, Ercoli L, Harris J, Heaton RK. Clinical and neuropsychological comparison of psychotic depression with nonpsychotic depression and schizophrenia. Am J Psychiatry 1996;153(4):490-6.

13. Burns BJ, Larson DB, Goldstrom ID, et al. Mental disorders among nursing home patients: preliminary findings from the National Nursing Home Survey pretest. Int J Geriatr Psychiatry 1988;3:27-35.

14. Schneider LS, Pollock VE, Lyness SA. A meta-analysis of controlled trials of neuroleptic treatment in dementia. J Am Geriatr Soc 1990;38:553-63.

15. Popli AP, Konicki PE, Jurjus GJ, et al. Clozapine and associated diabetes mellitus. J Clin Psychiatry 1997;58:108-11.

16. Bergemann N, Ehrig C, Diebold K, Mundt C, von Einsiedel R. Asymptomatic pancreatitis associated with clozapine. Pharmacopsychiatry 1999;32(2):78-80.

17. Katz IR, Jeste DV, Mintzer JE, et al. Comparison of risperidone and placebo for psychosis and behavioral disturbances associated with dementia: a randomized, double-blind trial. J Clin Psych 1999;60:107-15.

18. Petty RG. Prolactin and antipsychotic medications: mechanisms of action. Schizophr Res 1999;35(suppl):S67-S73.

19. Dickson RA, Glazer WM. Neuroleptic-induced hyperprolactinemia. Schizophr Res 1999;35(suppl):575-86.

20. Zarate CA, Baldessarini RJ, Siegel AJ, et al:. Risperidone in the elderly: a pharmacoepidemiological study. J Clin Psychiatry 1997;58:311-17.

21. Street JS, Clark WS, Gannon K, et al. Olanzapine treatment of psychotic and behavioral symptoms in patients with Alzheimer’s disease in nursing care facilities: a double-blind, placebo-controlled trial. Arch Gen Psychiatry 2000;57(10):968-76.

References

1. Hargreaves WA, Shumway M. Pharmacoeconomics of drug therapy. J Clin Psych 1996;57(suppl 9):66-76.

2. Lieberman J, Chakos M, Wu H, Alvir J, et al. Longitudinal study of brain morphology in first-episode schizophrenia. Biol Psych 2001;49(6):487-99.

3. Patterson TL, Shaw W, Semple SJ, et al. Health-related quality of life in older patients with schizophrenia and other psychoses: relationships among psychosocial and psychiatric factors. Int J Geriatr Psychiatry 1997;12(4):452-61.

4. Harris MJ, Jeste DV. Late-onset schizophrenia: an overview. Schizophr Bull 1988;14:39-55.

5. Kraeplin E. Dementia, praecox, and paraphrenia (1919). Translated by Barclay RM. Huntingdon NY: Krieger, 1971

6. Bleuler E. Late schizophrenic clinical pictures. Fortschr Neurol Psych 1943;15:259-90.

7. Pearlson G, Rabins P. The late-onset psychoses: possible risk factors. Psychiatr Clin North Am 1988;11(1):15-32.

8. Drevets WC, Rubin EH. Psychotic symptoms and the longitudinal course of senile dementia of the Alzheimer’s type. Biol Psychiatry 1989;25:39-48.

9. Wragg R, Jeste DV. Overview of depression and psychosis in Alzheimer’s disease. Am J Psychiatry 1989;146:577-87.

10. Lesser IM. Late-onset psychotic disorder not otherwise specified: clinical and neuroimaging findings. Biol Psychiatry 1992;31:419-23.

11. Lesser IM, Miller BL, Swartz JR, et al. Brain imaging in late-life schizophrenia and related psychoses. Schizophr Bull 1993;19:419-23.

12. Jeste DV, Heaton SC, Paulsen JS, Ercoli L, Harris J, Heaton RK. Clinical and neuropsychological comparison of psychotic depression with nonpsychotic depression and schizophrenia. Am J Psychiatry 1996;153(4):490-6.

13. Burns BJ, Larson DB, Goldstrom ID, et al. Mental disorders among nursing home patients: preliminary findings from the National Nursing Home Survey pretest. Int J Geriatr Psychiatry 1988;3:27-35.

14. Schneider LS, Pollock VE, Lyness SA. A meta-analysis of controlled trials of neuroleptic treatment in dementia. J Am Geriatr Soc 1990;38:553-63.

15. Popli AP, Konicki PE, Jurjus GJ, et al. Clozapine and associated diabetes mellitus. J Clin Psychiatry 1997;58:108-11.

16. Bergemann N, Ehrig C, Diebold K, Mundt C, von Einsiedel R. Asymptomatic pancreatitis associated with clozapine. Pharmacopsychiatry 1999;32(2):78-80.

17. Katz IR, Jeste DV, Mintzer JE, et al. Comparison of risperidone and placebo for psychosis and behavioral disturbances associated with dementia: a randomized, double-blind trial. J Clin Psych 1999;60:107-15.

18. Petty RG. Prolactin and antipsychotic medications: mechanisms of action. Schizophr Res 1999;35(suppl):S67-S73.

19. Dickson RA, Glazer WM. Neuroleptic-induced hyperprolactinemia. Schizophr Res 1999;35(suppl):575-86.

20. Zarate CA, Baldessarini RJ, Siegel AJ, et al:. Risperidone in the elderly: a pharmacoepidemiological study. J Clin Psychiatry 1997;58:311-17.

21. Street JS, Clark WS, Gannon K, et al. Olanzapine treatment of psychotic and behavioral symptoms in patients with Alzheimer’s disease in nursing care facilities: a double-blind, placebo-controlled trial. Arch Gen Psychiatry 2000;57(10):968-76.

Issue
Current Psychiatry - 01(07)
Issue
Current Psychiatry - 01(07)
Page Number
10-18
Page Number
10-18
Publications
Publications
Topics
Article Type
Display Headline
Late-life psychosis: It’s efficacy vs. cost in the tug-of-war over treatment
Display Headline
Late-life psychosis: It’s efficacy vs. cost in the tug-of-war over treatment
Sections
Article Source

PURLs Copyright

Inside the Article